Vordiplom Biopsychologie

Werbung
Kapitel 1
Nennen Sie 6 Teilgebiete der N.P. und beschreiben Sie diese kurz!

Physiologische Psychologie:
Erforschung der Auswirkungen von Veränderungen des Nervensystems auf das Verhalten unter
kontrollierten Laborbedingungen im Tierexperiment. Chirurgische, elektrische oder chemische
Eingriffe; streng kontrollierte experimentelle Bedingungen; Grundlagenforschung dominiert

Psychopharmakologie:
Die Erforschung pharmakologischer Einflüsse auf Gehirn und Verhalten. Meist angewandte
Forschung an Versuchstieren, wenn ethisch vertretbar auch Menschen

Neuropsychologie:
Erforschung der psychologischen Auswirkungen von Hirnschädigungen bei menschlichen
Patienten. Einflüsse von Hirnschäden auf menschliches Verhalten aus ethischen Gründen mit
Hilfe von Fallstudien von Patienten mit Hirnschäden.) Stark anwendungsorientiert.

Psychophysiologie:
Erforschung der Beziehung zwischen physiologischer Aktivität und psychologischen Prozessen
beim Menschen mit nichtinvasiven Beobachtungsmethoden. Forschung konzentriert sich auf
Verständnis der Physiologie bei psychologischen Prozessen, wie Aufmerksamkeit, Emotion und
Informationsverarbeitung; aber auch klinische Anwendungen.

Kognitive Neurowissenschaft
untersucht die neuronalen Grundlagen der Kognition, überwiegend an Menschen, nichtinvasiv,
mit bildgebenden Verfahren. Hohes Maß an Interdisziplinarität.

Vergleichende Psychologie
Betrachtet das Verhalten verschiedener Arten (Vergleich verschiedener Tierarten, einschließlich
Mensch) und konzentriert sich dabei auf die genetischen, evolutionsbiologischen und adaptive
Grundlagen des Verhaltens
Kapitel 2
Welche drei Faktoren bestimmen nach Auffassung der modernen biopsychologischen
Forschung das Verhalten?

Das Verhalten eines Individuums resultiert aus der aktuellen Situation, aus der Erfahrung des
Individuums und evolutionären Faktoren wie genetische Ausstattung.
Was versteht man unter Phrenologie?

Die Phrenologie ist eine zu Beginn des 19. Jahrhunderts von Josef Gall begründete
pseudowissenschaftliche Lehre, die versuchte geistige Eigenschaften und Zustände bestimmten,
klar abgegrenzten Hirnarealen zuzuordnen. Dabei wurde ein Zusammenhang zwischen Schädelund Gehirnform einerseits und Charakter und Geistesgaben andererseits unterstellt.
Was versteht man unter Präadapation?

Präadaption (Voranpassung) ist in der Entwicklungsbiologie das Vorhandensein eines zufällig
entstandenen Merkmales (z. Bsp. durch Mutation entstanden), das sich bei später veränderten
Umweltverhältnissen als nützlich erweist.
Nenne und erkläre 2 Verhaltensweisen, die in der Evolution eine Rolle spielen!

Soziale Dominanz
Bei vielen sozial lebenden Arten bauen die Männchen in wechselnden
Zweikämpfen eine stabile Rangordnung auf (Hierarchie sozialer Dominanz). Diese Hierarchie führt
zur Abnahme der Feindseligkeiten innerhalb der Gruppe. Die dominanten (Alpha) Männchen
kopulieren häufiger, als die in der Rangfolge unter ihnen stehenden Männchen und können daher
ihre Gene häufiger weitergeben.

Werbeverhalten
Bei vielen Arten geht der Paarung ein kompliziertes Werbeverhalten voraus. Das Männchen
signalisiert dem Weibchen sein Interesse durch chemische, akustische, optische oder taktile
Signale. Dies löst beim Weibchen seinerseits ein Signal aus, welches wiederum ein weiteres
Signal beim Männchen hervorruft, bis es zur Paarung kommt. Reagiert einer der beiden Partner
nicht oder falsch, unterbleibt die Paarung. Werbeverhalten fördert vermutlich die Evolution neuer
Arten
Die Evolution des Menschen ist eng an die Entwicklung des Hirns gebunden. Erklären Sie kurz
zwei Mechanismen , die zur drastischen Zunahme des Gehirnvolumens bei Homo erectus und
Homo sapiens geführt haben könnten. (2 Pkt.)

Funktionelle Spezialisierung:
Spezialisierung von Gehirnarealen (z.B. beim Klettern)

Neuentwicklung:
Zur Anpassung an Lebensraum (z.B. räumliches Sehen, Farbwahrnehmung)

Neotonie: infantile Erwachsene haben Möglichkeit größeres Gehirn zu entwickeln →
Infantilisierung der Kopfform

Präadaptation (Gehirngröße)
Was versteht man unter geschlechtsgebundenen Merkmalen? Welche Besonderheit weisen Sie
auf?


Geschlechtsgebundene Merkmale sind Merkmale, die von Genen auf dem
Geschlechtschromosom beeinflusst werden. Sie werden praktisch alle vom X Chromosom
kontrollier,t da das Y-Gen zu klein ist.
Besonders ist, dass sich dominante Merkmale häufiger beim weiblichen Geschlecht manifestieren,
rezessive Merkmale eher beim männlichen Geschlecht.
Was versteht man unter einem geschätzten Erblichkeitsgrad? Diskutieren Sie Probleme bei der
Erblichkeitsbestimmung am Beispiel der Intelligenz.



Eine Schätzung des Anteils der genetischen Varianz an der Gesamtvarianz, der für ein
bestimmtes Merkmal in einer bestimmten Untersuchung auftritt und aus der genetischen Variation
des Merkmals in dieser Untersuchung resultiert.
Genet. Unterschiede fördern psychologische Unterschiede, indem sie die Erfahrung beeinflussen
(welchen Lebensraum man sich aussucht) →geringe genetische Unterschiede führen eventuell zu
großen psychologischen Unterschieden, z.B. suchen sich Menschen mit dispositionaler Intelligenz
gleiche Erfahrungen, was die Intelligenz fördert, dies führt dazu, dass ein hoher Erblichkeitsgrad
bestimmt wird, obwohl die Umwelt einen sehr großen Einfluss hatte.
Außerdem nur Aussagen zu best. Zeitpunkt und best. Population →keine Generalisierung möglich
Genetische Studien haben gezeigt, dass genetische Vielfalt durch „crossing over“ erhöht wird.
Wie funktioniert dieses crossing over? In welcher Phase der Zellteilung tritt es auf?
1 Punkt


Nach der Verdoppelung des genetischen Materials legt sich ein Chromosom über das andere. An
der „Grossing-over-Stelle“ brechen die Chromosomen auseinander und tauschen Abschnitte aus.
Grossing over tritt während der Prophase I der Meiose auf
Nennen Sie die beiden Teilbereiche der „Genexpression“ und beschreiben Sie beide kurz!




Die Produktion eines Proteins nach der „Bauanweisung“ eines bestimmten Gens
Transkription eines DNA-Abschnitt auf einen RNA Basensequenzcode
Translation des RNA-Basensequenzcodes in eine Aminosäuresequenz.
Regulatorprotein (durch Signale aus Umwelt aktiviert) aktiviert Operatorgen → dieses aktiviert
Strukturgen (gibt Info zur Proteinsynthese) → DNA-Molekül öffnet sich →Transkription:
codierender Strang als Kopiervorlage für komplementären mRNA-Strang mit Uracil statt Thymin
→ mRNA bringt gen. Info ins Cytoplasma → Translation: Ende der mRNA bindet sich an Ribosom
→ dieses bewegt sich entlang der mRNA und liest gen. Info → 3 Basen (Codon) weisen Ribosom
an, best. Aminosäuren mittels tRNA aneinander zu hängen (Protein) → bis Stopp-Codon
Kapitel 3
Unterschied Dendriten und Axone?


Dendriten: kurze Fortsätze des Neurons, die Impulse zum Neuron hin leiten
Axone: lange Fortsätze, die Impulse vom Neuron fortleiten
Neurone werden häufig anhand der Fortsätze, die aus dem Zellkörper entspringen klassifiziert.
Welche Neuronentypen unterscheidet man dabei?




Unipolare Neuronen: einen Fortsatz
Bipolare Neuronen: zwei
Multipolare Neuronen: mehrere
Interneurone: sehr kurze Fortsätze in eng umschriebenem Gebiet
Nennen Sie die 3 Hauptfurchen und 2 Hauptwindungen (im Telencephalon)!


Hauptfurchen: Fissura longitudinalis, Fissura lateralis (Sylvische Furche), Sulcus centralis
Hauptwindungen: Gyrus praecentralis, Gyrus postcentralis, Gyrus temporalis superior, Gyrus
cinguli
Was zählt man zu den Basalganglien? Welche Funktionen werden ihnen zugeordnet?


Zu den Basalganglien gehören Amygdala, Nucleus caudatus und Putamen (Striatum), Globus
pallidus
motorisch Funktion und spielen eine entscheidende Rolle bei der Ausführung von
Willkürbewegungen
Nennen Sie drei Strukturen des Tegmentums Im Mittelhirn und Ihre Funktion

Substantia nigra


Nucleus ruber
Periaquaeductales Grau
Hat sensomotrische Funktionen, ist an der Planung und dem Beginn
einer Bewegung beteiligt
nimmt Einfluss auf den Muskeltonus und die Körperhaltung
Übermittlung analg. Wirkung von Opiaten
Welche Funktion hat das Limbische System? Nennen Sie vier wichtige Gehirnstrukturen im
Limbischen System.


Das limbische System ist bei der Steuerung von Emotionen und Motivationen beteiligt,
insbesondere bei der Auslösung von Flucht, Fressen, Kampf und Sexualverhalten.
Zum limbische System gehören die Mamillarkörpern, der Hippocampus, Amygdala, Gyrus cinguli,
Septum und Fornix
Wozu dienen Mitochondrien?

Ihre wichtigste Funktion ist Energiegewinnung durch oxidative Phosphorylierung bei der
Zellatmung. Sie wandeln Energie, die beim Umsatz der Nährstoffe mit Sauerstoff frei wird, in
Adenosintriphosphat (ATP) um.
Warum verfügen Neurone des peripheren – nicht aber des zentralen Nervensystems – über die
Fähigkeit zur Regeneration? 1 Punkt

Das periphere Nervensystem verfügt über Schwannzellen. Satellitenzellen, deren myelinreiche
Fortsätze sich um Axone im PNS wickeln. Neurotrophe Faktoren, die von Schwannzellen
freigesetzt werden, stimulieren das Wachstum neuer Axone, und die neuronalen
Zelladhäsionsmoleküle auf der Zellmembran der Schwann- Zellen schaffen Bahnen, auf denen
sich die ausgewachsene Axone vortasten können. Im zentralen Nervensystem gibt es keine
Schwannzellen. Daher dort keine neuronale Regeneration
Was bewirkt die Blut-Hirn-Schranke und wie kommt sie zustande?
 Durch die Blut-Hirn-Schranke wird verhindert, dass bestimmte toxische Substanzen aus dem Blut
ins Nervengewebe eindringen. Endothelzellen, mit denen Blutgefäße ausgekleidet sind, sind in
den Blutgefäßen des Gehirns eng miteinander verbunden und ohne jede Fensterung, dadurch
Schrankenwirkung. Wasserlösliche Stoffe und große Moleküle können nur passieren, wenn sie
an spezielle Rezeptormolekülen andocken können.
Nennen Sie 2 Teile des PNS!


somatischem Nervensystem
Der Teil, der mit der Umwelt interagiert. Es besteht aus afferenten Nerven, die sensorische
Informationen zum ZNS leitet und aus efferenten Nerven, die Signale aus dem ZNS an die
Skelettmuskulatur übermitteln.
vegetativem (autonomen) Nervensystem
Der Teil, der an der Regulierung des inneren Milieus beteiligt ist. Es besteht aus afferenten
Nerven, die Signale von den inneren Organen zum ZNS leiten und efferenten Nerven, die Signale
vom ZNS zu den inneren Organen übertragen.
Kapitel 4
Was versteht man unter einer Depolarisation bzw. einer Hyperpolarisation des
Membranpotentials?


Veränderung des Membranpotentials
Das Ruhemembranpotenzial beträgt -70 mV. Eine Verringerung um 30 mV auf -40 mV ist eine
Depolarisation (→EPSP: erhöht Feuerungswahrsch.). Eine Erhöhung um -20 mV auf -90 mV ist
eine Hyperpolarisation (IPSP: senkt Feuerungswahrsch.). Wird danach das
Ruhemembranpotenzial von -70 mV wieder erreicht, spricht man von Repolarisation
Was ist der Unterschied zwischen der absoluten und der relativen Refraktärzeit eines
Aktionspotentials? Wozu sind sie wichtig?




Ein Aktionspotential ist ein sehr schneller Aktivitätsausbruch eines einzelnen Neurons, bei dem
sich der Spannungsunterschied gegenüber der Umgebung in weniger als 1 ms um etwa 110 mV
ändert.
Absolute: Ein bis zwei Millisekunden nach Beginn des Aktionspotentials ist es einem Neuron nicht
möglich, ein weiteres Aktionspotential auszulösen, da alle Natrium-Kanäle geöffnet sind
Relative: danach folgt eine Periode in der die Erregungsschwelle für die Auslösung eines
Aktionspotentials höher liegt als normal, da alle Natrium-Kanäle inaktiv
Wichtig für:
Verhinderung einer Rückwärtsleitung des AP: AP nur in eine Richtung
Ein Neuron feuert mit einer Frequenz, die proportional zur Reizstärke ist. Bei langem und sehr
starken Reiz feuert es sobald die absolute Refraktärzeit vorbei ist, erneut.
Reicht die Reizintensität nur aus, um ein Neuron im Ruhezustand zum feuern zu bringen, lässt
sich ein weiteres AP erst nach Ablauf der relativen Refraktärzeit auslösen.
Erklären Sie, wie die Beseitigung von überflüssigen Neurotransmittern im synaptischen Spalt
abläuft.

Wiederaufnahme: häufigster deaktivierender Mechanismus. Neurotransmitter werden nach ihrer
Freisetzung in den synaptischen Spalt schnell wieder in die präsynaptischen Endknöpfe
aufgenommen. Sie werden dann erneut in Vesikel verpackt und wieder ausgeschüttet.
 Enzymatischer Abbau: Neuropeptide und Acetylcholin werden von Enzymen abgebaut.
Z. B. wird Acetylcholin mithilfe des Enzyms Acetylcholinesterase abgebaut.
Recycling: Viele der enzymatischen Abbauprodukte werden wieder vom präsynaptischen
Endknopf aufgenommen und zur Resynthese neuer Neurotransmittermoleküle benutzt.
Wodurch unterscheiden sich höhermolekulare Neurotransmitter von niedermolekularen in ihrer
Wirkungsweise? Rezeptor direkt, rezeptor gaba


Niedermolekulare Neurotransmitter (z.B. Monamine, Aminosäuren)
Meist Freisetzung von gerichteten Synapsen; Aktivierung ionotroper und metabotroper
Rezeptoren, die direkt auf Ionenkanäle wirken → Übermittlung kurzfristig erregender bzw.
hemmender Signale (EPSP bzw. IPSP → AP)
Höhermolekulare Neurotransmitter:
Meist diffuse Freisetzung; Aktivierung metabotroper Rezeptoren, die über second messenger
wirken →Übertragung langsamer, lang anhaltender Signale;
Neuropeptide, werden häufig als Neuromodulator bezeichnet, da sie nicht selbst Signale in
anderen Zellen induzieren, sondern die Empfindlichkeit von Neuronenpopulationen für die
erregenden oder hemmenden Signale schnell wirkender, gerichteter Synapsen reguliert.
Kapitel 5
Was ist bipolare und unipolare Ableitung?


Bipolar: Berechnung der Differenz zw. zwei Elektroden an aktiven Stellen
Unipolar: Berechnung der Diff. Zw. einer Elektrode an aktiver Stelle und Referenz an neutraler
Stelle
Definieren Sie Ableitung, Biosignal und Parameter/Indikator!



Unter Ableitung versteht man die Messung von Biosignalen
Biosignale sind am Körper gemessene Phänomene wie Pulsfrequenz oder Atemrhythmus
Parameter sind Kennwerte, die einen Teilaspekt des Biosignals darstellen, der in Zusammenhang
mit psychologischen Phänomenen gebracht wird (z.B. Spannungschwankungen)
Was sind Artefakte?


Einflüsse biolog. oder techn. Herkunft, die uns nicht interessieren (Tagesschwankungen,
Bewegungen,…) → müssen kontrolliert werden
Z.B. Augenbewegungen (→Polarisation der Retina) → emp. Ermittlung des EOG, um EEG zu
korrigieren
Was versteht man unter einem AEP? Welche Charakteristikea weist es auf?
Akustisch-evoziertes-Potential: Die ersten hundert ms des EKPs auf auditorische Reize
Frühe Peaks (bis 10 ms) werden zwischen akustischen Nerv und CGL
generiert. Negative (N) und positive Komponenten (P) repräsentieren Aktivität aus dem Thalamus
und auditiven Kortex.
Welche Frequenzbänder weist das menschliche EEG auf? Durch welchen
Frequenzbereich sind sie charakterisiert?
Übergange fließend bzgl. der Gestalt und der funktionellen Bedeutung
- Alpha
8-13Hz
5-100 µV
okzipital, parietal
- Beta
14-30Hz
2-20 µV
präzentral, frontal
- Theta
5-7Hz
5-100 µV
frontal, temporal
- Delta
0,5-4Hz
20-200µV
variabel
- Gamma:
> 40 Hz
entspannter Wachzustand
mental u. körp. aktiv
Übergang zum Einschlafen
Tiefschlaf
kogn. Aktivität
Was versteht man unter einem ereigniskorrelierten Potential?




Veränderung der elektr. Aktivität des Gehirns
typische Potentialverläufe, die auf best. äußere und innere Reize folgen (teilweise schon vorher)
sie sind wiederholbar und zeigen die gleichen zeitlichen Charakteristiken
müssen gemittelt werden, um Rauschen (Spontan EEG und Umwelteinflüsse) rauszurechnen
(Signal-Rauschen-Verhältnis = Wurzel aus Anzahl der Mittelungen)
Was versteht man unter einem evoziertem Potential?

Eine Veränderung der elektrischen Aktivität des Gehirns, die durch die Präsentation eines
sensorischen Stimulus hervorgerufen wird. Ein evoziertes Potential ist ein spezieller Typ des
ereigniskorrelierten Potentials.

Es entsteht durch synchrones Auftreten postsynapt. Potentiale größerer Zellverbände (allg.
EEG)
Was versteht man unter einem elektrischen Dipol? Nennen Sie ein Beispiel!

Ein elektrischer Dipol besteht aus zwei gleich großen Ladungen unterschiedlichen
Vorzeichens (+,-). Ein einfaches Beispiel für einen Dipol ist ein Stabmagnet.

Neurologisch: Ein postsynaptisches Potential verändert das Ladungsgefüge an einem Neuron und
führt so zum Aufbau eines elektrischen Feldes.
1) Axosomatische V.: bei exzitatorischer Wirkung → Depolarisation des Zellkörpers
→ extrazelluläre Umgebung wird negativer → Umgebung der Endigung der
Apikaldendriten (nahe Kortexoberfläche) wird positiver
2) Axodendritische V.: bei exzitatorischer Wirkung → extrazelluläre Umgebung
(Kortexoberfläche) neg. → Umgebung des Zellkörpers pos.
→ wirken Synapsen inhibitorisch, kehren sich Potentialverhältnisse um
Was versteht man unter exogenen Komponenten des ereigniskorrelierten Potentials?

Unter exogenen Komponenten versteht man die Komponenten eines ereigniskorrelierten
Potentials die auf äußere Stimuli zurückzuführen sind. Exogene Komponenten sind allein von den
physikalischen Reizeigenschaften (z.B. Intensität) abhängig. Exogene Potentiale sind durch
psychologische Variablen nicht veränderbar, nicht durch Aufmerksamkeit oder die
Gedächtnisspanne (usw.), aber auch nicht durch allgemeine organismische Zustände wie etwa
dem Schlaf. Eindeutig exogen sind nur die frühen Potentiale (unter 100ms)
Was versteht man unter endogenen Komponenten des ereigniskorrelierten Potentials?

Unter endogenen Komponenten versteht man die Komponenten eines ereigniskorrelierten
Potentials die mit höheren kognitiven Prozessen und einer internen Verarbeitung assoziiert sind.
Endogene Potentiale spiegeln psychologische Prozesse wider, sie sind von physikalischen
Reizeigenschaften unabhängig; sie sind auch unabhängig von der Modalität, auf der ein Reiz
dargeboten wird. Endogene Anteile finden sich ab ca. 100 ms
Was versteht man unter dem „N100-Aufmerksamkeitseffekt“? Beschreiben Sie eine
experimentelle Anordnung mit der er sich untersuchen lässt.

N100 = Komponente des visuellen EKP, neg. Peak bei 140-190 ms nach On-Set
Verstärkung von Stimuli, die in beachteten Lokationen gezeigt werden → N100-Amplitude ist
größer für beachtete Stimuli, verglichen mit Stimuli, die nicht beachtet werden sollen
Versuchsanordnung: Vpn fixiert Kreuz in Mitte des Bildschirms → Anweisung nur auf Stimuli zu
achten, die auf der linken Seite aufblinken und diejenigen zu ignorieren, die rechts aufblinken
Was versteht man unter dem „P200-Aufmerksamkeitseffekt“? Beschreiben Sie eine
experimentelle Anordnung mit der er sich untersuchen lässt.

Die positive EEG-Welle, die gewöhnlich rund 200 Millisekunden nach einem bestimmten Stimulus
auftritt fffffffffffff
Was versteht man unter dem „P300-Aufmerksamkeitseffekt“? Beschreiben Sie eine
experimentelle Anordnung mit der er sich untersuchen lässt.

Die positive EEG-Welle, die gewöhnlich rund 300 Millisekunden nach einem bestimmten Stimulus
auftritt, der für den Probanden beziehungsweise das Versuchstier von Bedeutung ist. Die P300
wird üblicherweise mit dem klassischen „Oddball-Paradigma“ evoziert. Das Oddball-Paradigma
ist dadurch gekennzeichnet, dass innerhalb einer Serie gleichbleibender Reize einige
abweichende Reize in zufälliger Reihenfolge mit geringer Wahrscheinlichkeit dargeboten werden.
Die VP hört eine Reihe von Tönen in einem bestimmten Muster. (Höhe, Frequenz…) Kommt ein
unerwarteter Ton, tritt der P300-Effekt auf → widerspiegelung von Überraschung (auch
unerwartetes Ausbleiben eines Stimulus)
Was ist die N400? Experimentelle Anordnung?

Bedeutungsverarbeitung (nicht an Sprache gebunden): semantische Anomalien generieren große
N400 („Ich trinke meinen Kaffe mit Zucker / Socken“).

Verschiedene Schriftart → große P560
Was ist die CNV?




Endogene Komponente des EKP: Contingent Negative Variation
Sich langsam aufbauende neg. Welle
Vorbereitung auf ein bevorstehendes Ereignis (kein Bereitschaftspotential)
Z.B. bei zwei verknüpften Reizen
Was ist ein Bereitschaftspotential?

Ein Bereitschaftspotential ist ein negatives elektrisches Potential, das aufgrund von neuronalen
Aktivitäten vor allem im supplementär motorischen Areal beider Hirnhälften auftritt und in engem
zeitlichen Zusammenhang mit der Einleitung willkürlicher Bewegungen steht. Das
Bereitschaftspotential zeigt sich im EEG durchschnittlich 550 Millisekunden vor einer
Willkürbewegung, der bewusste Wille zur Bewegung wird von den Versuchspersonen jedoch im
Durchschnitt erst 200 Millisekunden vor der Bewegung angegeben.
Was versteht man unter Mismatch negativity?

Präsentiert man das Oddball-Paradigma mit auditiven Stimuli und organisiert den Testablauf so,
daß die Probanden die Stimuli ignorieren, indem z.B. ein (stummer) Videofilm gezeigt wird dann
lassen sich für den 'Standard'-Reiz und für den 'Devianten' im Zeitbereich von circa 100 - 250
msec nach Stimulusonset jeweils unterschiedliche negative Auslenkungen im EEG beobachten.
Die Differenz zwischen beiden Auslenkungen nennt man die mismatch negativity
→ unabhängig von Aufmerksamkeit
Was versteht man unter Processing negativity?

Hält man Versuchspersonen an, bei aufeinanderfolgenden Stimuli auf bestimmte
Stimuluseigenschaften zu achten, dann entsteht statt der mismatch-negativity eine negative
Auslenkung im Zeitfenster von etwa 80 msec bis zu gegebenenfalls 300 msec nach Stimulusonset. Dabei ist für die Auslösung einer processing-negativity entscheidend, daß im Strom der
Signale solche mit einer spezifischen Eigenschaft beachtet werden, wohingegen die anderen
Signale unbeachtet bleiben. Die Differenz der Auslenkung einerseits bei beachteten Signalen und
andererseits bei den unbeachteten Signalen ergibt das Differenzpotential der processing
negativity. → abhängig von Aufmerksamkeit
Wie funktioniert MRT?




Erfassung der Dichte und Schwingungseigenschaften magnetisch erregter Wasserstoffkerne
(Protonen)
Im Magnetfeld richten sich pos. Protone in Nord-Süd-Richtung aus. Die Einstreuung
elektromagnetischer Impulse rechtwinklig zur Richtung des Magnetfeldes führt dazu, dass sich die
Protonen wie Kreisel bewegen. Das Abstellen der Impulse bewirkt, dass die Kerne
zurückschwingen.
Relaxationszeiten: T1: Zeit, um wieder in Ausgangsposition zu schwingen; T2: Abnahme des
Drehimpuls → Zeit bis sie sich nicht mehr drehen
Aus untersch. Zeitkonstanten berechnet man Gehirnbilder
Der BOLD-Effekt bildet die Grundlage der funktionellen MRT. Was versteht man unter BOLDEffekt? (2 Pkt.)

BOLD steht für Blood Oxygen Level Depended-Effekt: hämodynamisches Korrelat zu EKP;
allerdings erst 5s später messbar

Erhöhter zellulärer Energiebedarf bei neuronaler Aktivierung führt zur Zunahme
sauerstoffarmen (desoxy.) Hämoglobins → Blutflußzunahme größer als Sauerstoffzufuhr
→ Abbau von desoxy. HG führt zu langsameren T2 Relaxationszeiten →
Konzentrationsänderung des desoxy. HG ruft Magnetfeldinhomogenitäten hervor
BOLD-Aktivierung muss zu .95 mit Stimulusmuster korrelieren, sonst nicht reliabel
(Rauschen)

Wie funktioniert PET?


Positronen-Emissions-Tomographie: isotope (schwach radioaktiv) werden in Blutbahn
gegeben → dort werden sie genauso behandelt wie ein Sauerstoffatom → Kollision von
Positron und Elektron → Gammastrahlung wird gemessen →Rückrechnung, wann Kollision
stattfand → welche Region verstoffwechselt mehr Sauerstoff bei best. Reizen
Subtraktionslogik: betrachtet wird die Differenz zw. Stimulus- und Kontrollbedingung
Nennen Sie zwei Vorteile, die die funktionelle Magnet-Resonanz-Tomographie gegenüber der
Positronen-Emissions-Tomographie aufweist.


Die funktionelle Kernspin- oder fMR –Tomographie (functional magnetic resonance)
macht die erhöhte Sauerstoffversorgung (infolge der gesteigerten Durchblutung) der
aktiven Hirngebiete sichtbar.
Diese Technik weist gegenüber der PET vier Vorteile auf:
1)
2)
3)
4)
Dem Patienten muss nichts injiziert werden,
das fMR Tomographie Bild liefert gleichzeitig strukturelle und funktionelle Informationen,
die räumliche Auflösung ist besser, und
sie kann zur Erstellung dreidimensionaler Bilder verwendet werden.
Unterschied bildgebender Verfahren und EEG/MEG?



Neurovaskuläre Kopplung: Stimulus → neuronale Antwort →? →vaskuläre Antwort
(Sauerstoffzufuhr)
Bildgebende Verfahren (PET/fMRT): Umweg über Metabolismus (vaskuläre Antwort) zum
Abbilden der Hirnfunktion
EEG/MEG: direkte Ableitung der neuronalen Antwort
Was besagt das Sampling-Theorem?



Beim Samplen (Analog-Digital-Wandlung) entsteht eine neue Frequenz, d.h. es handelt sich nicht
um gleichwertige Info wie die zuvor
Das Signal lässt sich immer dann komplett rekonstruieren, wenn es keine Frequenzen hat, die
oberhalb der Hälfte der Digitalisierungsfrequenz (Abtastrate: wie viele Messwerte pro Sek.) liegt
Bsp.: bei einem Signal von 9 Hz braucht man eine Abtastrate von mind. 18 Hz
Wie funktioniert EROS?
Optical imaging: event related optical signal
Ionenveränderung an Membran führt zu Veränderung der Lichtabsorption des Gewebes →
Bestrahlung mit Licht → Messung des austretenden Lichts → Vergleich der beiden Lichtstärken
Vorteile: billig, unschädlich, sensitiv gegenüber reduzierter vaskulärer Aktivität (Alzheimer), auch bei
hohem Sauerstofftransport ändert sich Brechungseigenschaft des Lichts
Kapitel 6
Was versteht man unter einer cerebralen Hämorrhagie? Wie kommt sie zustande?


Eine cerebrale Hämorrhagie ist eine Gehirnblutung. Sie tritt auf, wenn ein Blutgefäß im Gehirn
reißt, Blut in die das umliegende Nervengewebe sickert und es schädigt.
Häufige Ursache: Aneurysma: ballonartige Erweiterung in der Wand eines Blutgefäßes
Folge von Ischämie?

Gluatmat-Kaskade: exzessive Glutamatfreisetzung → Aktivierung von NMDA-Rezeptoren →
Einfluss von Natrium- und Calciumionen auf postsynapt. Neuron → Absterben des Neurons mit
gleichzeitiger Auslösung exzessiver Glutamat-Ausschüttung
Was versteht man unter „cerebrovaskulären Störungen“? Welche 3 Unterklassen gibt es?



Cerebrovaskuläre Störungen (Schlaganfälle) sind Störungen des Gehirnblutstoffwechsels, die
einen Hirnschaden verursachen → Infarkt: abgestorbenes Gewebe aufgrund eines Schlaganfalls
cerebrale Hämorrhagie
Hirnblutung
cerebrale Ischämie
Mangeldurchblutung durch
Thrombose
Verstopfte Vene
Embolie
Thrombose im Gehirn
Arteriosklerose
Gefäßverengung im Gehirn aufgrund von Fettablagerungen
Der Schlaganfall ist die dritthäufigste Todesursache in den USA. Was versteht man unter
einem Schlaganfall? Welche Haupttypen unterscheidet man und wie lassen sie sich
charakterisieren?



Ein Schlaganfall ist eine plötzlich eintretende cerebrovaskuläre Störung die zu Hirnschäden führt.
Unterschieden werden dabei:
Hämorragie (Einblutung)
Ein Blutgefäß im Hirn reißt und Blut sickert in das umliegende Nervengewebe und schädigt dies
Ischämie (Mangeldurchblutung)
Hierbei wird die Blutzufuhr zu einer Hirnregion unterbrochen (durch Thrombose, Arteriosklerose
oder einer Embolie), es kommt zu einer Unterversorgung mit Sauerstoff. Der unterversorgte
Bereich stirbt ab.
Was ist Angiographie?
Methode zur Sichtbarmachung der Blutgefäße im Gehirn
Wie entstehen gedeckte Schädel-Hirn-Traumata? Welche gibt es?


Durch starke Gewalteinwirkung auf Gehirn
Hirnquetschungen, Hämatome, Contre-Coup-Verletzungen (Hämatome auf kontralateraler Seite
des eigentlichen Stoßes aufgrund von Trägheit, häufig reversibel), Gehirnerschütterung
(Commotio cerebri, ohne Quetschungen) → Punch-Drunk-Syndrom
Was ist ein Punch-drunk Syndrom?

Demenz und Gehirnvernarbungen, durch wiederholte Stöße gegen den Kopf. Tritt häufig bei
Boxern auf. Zeigt, dass sich einzelner Gehirnerschütterungen im Laufe der Zeit zu schweren
Schäden aufsummieren können.
Nennen Sie Ursachen für Gehirnerkrankungen!
 Gehirninfektionen
 Neurotoxine
 Genetische Faktoren
 Programmierter Zelltod (Apoptose)
Nenne eine Gehirninfektion. Wie entsteht sie?



Encephalitis (Gehirnentzündung) aufgrund
Bakterieninfektionen (Meningitis, Hirnabszesse, Syphilis, progressive Paralyse)
Virusinfektionen (neurotrope Viren: greifen Gewebe an (Tollwut); pantrop: im ganzen Körper
(Mumps); Prionen: Eiweißmoleküle stören Gehirnstoffwechsel (BSE, Creutzfeld-Jakob))
Was sind Neurotoxine? Und was können sie bewirken?




Nervengifte, die durch Lunge oder Haut ins Blut gelangen → Durchdringen der Blut-HirnSchranke
Toxische Psychosen durch Schwermetalle
Spätdyskinesie durch Antipsychotische Medikamente
Korsakow-Syndrom durch Alkohol/Thiaminmangel
Genetische Faktoren? Beispiele?



Chromosomenaberration → Down-Syndrom
Abnorme rezessive Gene → Phenylketonurie
Abnorme dominante Gene → Huntington-Krankheit
Nennen Sie zwei Mechanismen des Zelltodes


 Apoptose: genet. programmiertes Schrumpfen, Eliminieren des Zellkerns durch Nachbarzellen
Zellnekrose: entzündliche Prozesse
Beide Formen bei Ischämie (nach Zellnekrose kommt Apoptose)
Nennen Sie neurologische Erkrankungen mit psycholog. Störungen!





Epilepsie
Parkinson
Huntington
Multiple Sklerose
Alzheimer
Was ist „Multiple Sklerose“? Nennen Sie die Symptome!


Multiple Sklerose ist eine Autoimmunerkrankung die zur allmählichen Zerstörung der
Myelinscheiden führt
die Symptome sind Harninkontinenz, Sehstörungen, Muskelschwäche, Taubheit der Gliedmaßen,
Intensionstremor und Ataxie (Verlust der Bewegungskoordination).
Was versteht man unter einer Autoimmunerkrankung? Nennen Sie ein Beispiel.


Autoimmunerkrankungen sind Krankheiten, bei denen das Immunsystem körpereigenes Gewebe
irrtümlicherweise als einen zu bekämpfenden Fremdkörper erkennt. Dadurch kommt es zu
schweren Entzündungsreaktionen, die zu Schäden am betroffenen Gewebe führen.
Ein Beispiel ist Multiple Sklerose bei denen die Myelinscheiden angegriffen werden.
Zu welcher Gehirnregion gehört die Substantia nigra? Wo liegt sie? Welche Krankheit tritt bei
Schädigung auf?




Die Substantia Nigra ist in der Region des Mesencephalons (Mittelhirns) zu finden.
Sie liegt im Tegmentum, dem gesamten Areal ventral des Tectums.
Bei Schädigung der substantia nigra kommt es zur parkinsonschen Krankheit, weil im Striatum ein
Dopaminmangel herrscht (nigrastratiale Bahn)
Symptome: Ruhetremor, maskenartiges Gesicht, Bewegungsverlangsamung, keine intellektuellen
Beeinträchtigungen
Nennen Sie Symptome und Ursache der Huntington-Krankheit!


Prorea auch genannt: progressive Bewegungsstörung (Veiztanz), seltener Demenz
Ursache: dominantes Gen → spätes Auftreten → Degeneration des Striatums, Verschwammung
der Großhirnrinde
Es gibt 2 verschiedene Arten von Epilepsieanfällen, partielle und generalisierte Anfälle.
Nennen Sie die beiden Unterklassen der partiellen Anfälle und beschreiben Sie diese kurz!

Einfache Partielle Anfälle treten nur in einem Teil des Gehirns auf. Die Symptome sind
überwiegend sensorischer/motorischer Natur.

komplexen partiellen Anfällen, beschränken sich gewöhnlich auf den Temporallappen
(hierarchisch höher gelegene Hirnregionen), beginnen mit einer Aura und gehen mit
Bewusstseinsstörungen und psychomotorischen Attacken einher.
Nennen Sie die beiden Unterklassen der generalisierten Anfälle und beschreiben Sie diese
kurz!

Grand-mal-Anfall: der Anfall betrifft das gesamte Gehirn (hypersynchrone Aktivität) und entsteht
entweder Herdförmig oder setzt im gesamten Hirn gleichzeitig ein. Die primären Symptome sind
Bewusstlosigkeit, Verlust des Gleichgewichts und tonisch-klonische Krämpfe.

Petit-mal-Anfall: betrifft auch das gesamte Gehirn. Jedoch kommt es nicht immer zu Krämpfen.
Wichtigstes Symptom ist die Absence, eine Art von Bewusstseinstrübung
Beschreiben Sie das EEG Muster während eines Petit-mal Anfalls.

Das EEG zeigt eine charakteristische Abfolge spitzer und langsamer Krampfwellen, ein bilateral
symmetrisches spike-and-wave-Muster mit einer Frequenz von etwa 3Hz
Alzheimer-Krankheit




Gedächtnisschwäche, Sprachstörungen, Depressionen → im fortgeschrittenen Stadium: alle
Symptome einer Demenz
Nur in Form von Autopsie festzustellen → Diagnose ausschließlich auf Verhaltensebene
Verantw. Gene vermutlich auf Chromosom 21,14,19
Typ. Neurolog. Befunde: Neurofibrillenknäuel, Amyloidplaques (Klumpen degenerierter Neurone),
Neuronenverlust in Regionen höherer kogn. Funktion
Nenne drei Arten von Tiermodellen!



Homolog: Versuch, eine Erkrankung im Tier zu replizieren und dann zu behandeln (KindlingModell der Epilepsie)
Isomorph: ähnliche spontane Phänomene beim Tier wie beim Mensch, aber andere Ursache
Prädiktiv: Reaktionsverlauf best. Erkrankungen im Tiermodell untersuchen
Kapitel 7
Zellulärer Aufbau der Retina

Ganglienzellen → Amakrinen → Bipolarzellen → Horizontalzellen → Zapfen/Stäbchen
Unterschiede zw. Stäbchen und Zapfen



Stäbchen: häufiger; an Peripherie der Retina; hohe Lichtsensibilität → Sehen bei ungünstigem
Licht, aber keine Details und schlechtere Farbwahrnehmung; skotopisch; hohe Konvergenz
(mehrere Stäbchen projezieren auf eine Ganglienzelle → Aufsummieren der Aktivität, aber
geringere Sehschärfe)
Zapfen: wenige; v.a. an Fovea centralis; niedrige Lichtsensibilität → höhere Detail- und
Farbwahrnehmung; photopisch; geringe Konvergenz (1 zu 1 Verschaltung mit Ganglienzelle →
hohe räumliche Auflösung)
Duplizitätstheorie: Zapfen und Stäbchen sind Rezeptoren zweier versch. Systeme: eines für
skoptisches (schwarz-weiß, geringe Auflösung, hohe Sensitivität) und eines für photopisches
Sehen (farbig, hohe Auflösung, nur bei ausreichend Lichtintensität)
Was ist der Purkinje-Effekt? Wie kommt er zustande?


Bei intensivem Licht erscheint langwelliges Licht (rot, gelb) heller als kurzwelliges (blau, grün) bei
gleicher Intensität; in dämmrigem Licht umgekehrt
Er beruht auf der unterschiedlichen spektralen Empfindlichkeit der Sehzellen bei Tag- und
Nachtsehen. Am Tag sind vor allem die farbempfindlichen Zapfen aktiv, in der Nacht vor allem die
lichtempfindlichen Stäbchen.
Was ist Querdisparation? Wofür ist sie wichtig?

Netzhautbilder sind nie völlig identisch → aus Differenz wird Entfernung geschätzt
Nennen Sie zwei Baufehler der Retina!


Stäbchen und Zapfen (Rezeptoren) an Hinterseite der Retina → Kompensation durch Fovea
centralis: Stelle des schärfsten Sehens, wenig Ganglienzellen →Lichteinfall wird nicht gebrochen,
ausschließlich Zapfen
Blinder Fleck: wo Sehnerv Auge verlässt, keine Rezeptoren → einzelne Punkte verschwinden,
Balken jedoch nicht unterbrochen → Gehirn kompensiert fehlenden Wahrnehmungseindruck
(Ergänzungseffekt)
Beschreiben Sie den Weg der visuellen Informationen im Gehirn!

Zentrale Bahn (Retino-geniculo-striäre Bahn): Retina → Nervus opticus → Chiasma opticum
(ipsilaterale Weiterleitung vom temporalen Teil der Retina, kontralaterale vom nasalen Teil) →
Tractus opticus → Corpus geniculatum laterale im Thalamus → primärer visueller Cortex (auch
Area striata, Streifenfeld, Sehrinde) (BA 17, Occipitallappen)
Was versteht man unter retinotroper Organisation der zentralen Sehbahn?

Sie entspricht auf jeder Ebene einer Landkarte der Retina, d.h. zwei Stimuli, die auf
benachbarte Stellen in der Retina fallen, erregn auch auf allen Ebenen des visuellen Systems
benachbarte Neurone. Fovea centralis ist übergroß repräsentiert.
Hubel und Wiesel entdeckten, dass der primäre visuelle Cortex eine säulenhafte Struktur
aufweist. Erläutern Sie in Stichworten dieses wichtige Organisationsprinzip des visuellen
Cortex.




Liegt an der hinteren Spitze des Okzipitallappens (Brodman-Areal 17)
Neurone des visuellen Cortex sind funktionell in vertikale Säulen angeordnet
Rezeptive Felder der untereinander liegenden Zellen derselben Säule liegen im gleichen Bereich
des Gesichtsfeldes → die Fläche des Gesichtsfeldes, die von allen Zellen einer Säule überdeckt
wird, heißt aggregiertes Feld.
Diese Säulen liegen in einem Cluster zusammen: dabei erhält jeweils eine Hälfte Signale vom
linken, die andere vom rechten Auge



Positionssäulen
Die Cortexneuronen sind retinotop angeordnet, ihre Positionen im Cortex stimmen also mit den
Positionen ihrer rezeptiven Felder auf der Netzhaut überein.
Orientierungssäulen
Komplexe Zellen werden durch den Input mehrerer einfacher Zellen mit der selben
Orientierungspräferenz und endinhibierte Zellen durch den Input mehrerer komplexer Zellen mit
der selben Orientierungspräferenz konstruiert. Zellen schließen sich deshalb zu Gruppen mit
ähnlicher Orientierungspräferenz zusammen, dass dieser Konstruktionsprozess leichter
vonstatten geht.
Augendominanzsäulen
Der Cortex besteht aus einer Reihe von Säulen, deren Augendominanz (ca. 80% der
Cortexneurone reagieren sowohl auf die Reizung des rechten, als auch des linken Auges; die
meisten Zellen sprechen jedoch besser auf ein Auge als auf das Andere an) immer von links nach
rechts wechselt.
Was versteht man unter einem „rezeptiven Feld“? Beschreiben Sie die Funktionsweise
rezeptiver Felder anhand eines Beispiels aus dem Bereich visueller Neurone.
Zusatz: Wie unterscheiden sich Zellen des visuellen Cortex bzgl. Ihrer rezeptiven Felder?




Der Bereich innerhalb dessen ein geeigneter Reiz die Impulsfrequenz eines Neurons
beeinflussen kann (z.B. auf der Retina)
In der Retina finden sich konzentrische Zentrum-Peripherie antagonistische rezeptive Felder, die
entweder on-Zentrum off-Peripherie oder umgekehrt reagieren.
Z. B. reagieren On-Zentrum-Neurone auf Lichtreize in ihrer zentralen Region ihrer Felder mit
einer On-Reaktion und mit einer Off-Reaktion, wenn die peripheren Felder bestrahlt werden
(reagieren am stärksten auf Kontrast)
On-Zentrum und Off-Zentrum-Neurone (Ausnahme)
Einfache Zellen: reagieren optimal auf geradkantige Reize in einer best. Position und
Orientierung; monokular (reagieren nur auf Stimulation eines best. Auges)
Komplexe Zellen: reagieren optimal auf in best. Weise ausgerichtete gerade Kanten, unabh. von
Position der Kante innerhalb des rezeptiven Feldes (best. Bewegungsrichtung); viele sind
binokular
Was ist Phototransduktion? Welches Pigment spielt dabei eine Rolle?

Phototransduktion ist die Umwandlung von Lichtreizen in chemische und danach in
elektrische Impulse, die dann über den Sehnerv ins Gehirn weitergeleitet werden. Rhodopsin
spielt dabei eine wichtige Rolle. Trifft Licht auf das Pigment, verändert es seine Form und löst
dann einen elektrischen Impuls aus.
Welche Bereiche zählen zum visuellen System?

Zentrale Sehbahn




Visuelle Felder der Großhirnrinde:
Primärer visueller Kortex (striärer Kortex)
Posterior parietaler Kortex (Assoziationskortex)
Prästriärer Kortex
Gyrus temporalis inferior
 Kapitel 8
Was versteht man unter einem primären sensorischen Cortex. Über wie viele verfügt unser
Gehirn? Wo liegen sie?

Unter einem primären sensorischen Cortex versteht man das Cortexgebiet eines Sinnesbereichs,
das die meisten seiner Zuflüsse direkt aus den Umschaltstationen dieses Systems in den
Thalamuskernen erhält.




Unser Gehirn verfügt über 5 primäre sensorische cortices.
Primärer visueller C.
BA 17; an der hinteren Spitze des Okzipitallappens
Primärer auditorischer C.
BA 41,42; in der Sylvischen Furche/ Fissura Lateralis
Primärer somatosensorischer C. BA 1,2,3; Gyrus postcentralis des Parietallappen


Primärer motorischer C.
Primärer gustator. C.
BA 4; Gyrus praecentralis des Frontallappens
sylvische Furche, in der Nähe des Gesichtbereichs des
somatosens. Homunculus
Was versteht man unter einem sekundären sensorischen Cortex.

Unter einem sekundären sensorischen Cortex versteht man Cortexgebiete eines Sinnesbereichs,
die einen Großteil ihres Inputs vom primären sensorischen Kortex erhalten und von anderen
Bereichen des sek. Kortex desselben Sinnessystems
Beschreibe das frühere und das heutige Modell zur Organisation sensorischer Systeme!

Rezeptoren → Thalamus → primärer sensorischer Kortex (bekommt die meisten seiner Zuflüsse
direkt aus Umschaltstationen im Thalamus) → sekundärere sensorischer Kortex (die meisten
Signale aus primärem K. und von anderen Bereichen des sekundären K.) → Assoziationskortex
(Signale von mehr als einem Sinnessystem)

Früher: hierarchische Organisation (Nervenzellen immer anspruchsvoller hinsichtlich
Input); funktionelle einheitlich; serielle Verarbeitung

Heute: hierarchisch, funktionell untergliedert, parallel aufgebaut, zentrifugale Bahnen
(absteigende Bahnen, die Info von höheren auf darunter liegende Hierarchieebenen zurückliefern)
Was ist ein Skotom? Was eine Hemianopsie? Mit welcher Technik diagnostiziert?




Skotom: selektive Beeinträchtigung der Wahrnehmung in best. Bereichen des Gesichtsfeldes
Hemianopsie: kompletter Ausfall einer Hälfte
Perimetrische Bestimmung des Gesichtsfeldes: Lichtpunkt fixieren mit einem Auge zu →
kleinerer Lichtpunkt bewegt sich vom Rand des Bildschirms zur Mitte → Patient drückt Knopf,
wenn er ihn sieht
Ergänzungseffekt: Teil des Bildes, der im Skotom liegt wird häufig vom visuellen System
vervollständigt
Was versteht man unter „Blindsehen“?


Blindsehen ist die Fähigkeit von cortical blinden Patienten (mit Skotom), visuelle Aufgaben zu
lösen(etwa nach einem Objekt greifen), ohne dass sie sich des Sehens bewusst sind.
Patient D.B.: chirurgische Entfernung des rechten Occipitallappens → Erblindung des linken
Sehfeldes
Neuronale Erkrankungen des visuellen Systems machen sich meist durch spezifische
Beeinträchtigungen in nur einem sensorischen System bemerkbar. Erläutern Sie anhand eines
Beispiels, was man unter einer „visuellen Agnosie“ versteht?

Unter visueller Agnosie versteht man die Unfähigkeit zu Objekterkennung. Die Unfähigkeit geht
nicht auf eine Störung der sensorischen, verbalen oder intellektuellen Leistungen zurück. Bei der
visuellen Agnosie können Patienten die optischen Reize zwar sehen, wissen aber nicht worum es
sich handelt.

Ein Beispiel ist die Prosopagnosie:
Patienten haben Schwierigkeiten Gesichter zu unterscheiden bzw. zu erkennen z.T.. Sie haben
jedoch keine Probleme Gesichter als solche zu erkennen und einzelne Bestandteile (Nase) zu
identifizieren.
Von wo nach wo verläuft die Ventralbahn? Für welche Informationen ist sie spezialisiert?


Die Ventralbahn läuft von der primären Sehrinde über den ventralen prästriären Cortex zum
Gyrus temporalis Inferior.
Es gibt zwei Theorien: 1. „was-Bahn“, die Wahrnehmung, um was es sich bei dem Objekt handelt
oder 2. zuständig für bewusste Wahrnehmung
Von wo nach wo verläuft die Dorsalbahn? Für welche Informationen ist sie spezialisiert?


Die Dorsalbahn läuft von der primären Sehrinde über den dorsalen prästriären Cortex zum
posterioren parietalen Cortex
2 Theorien: 1. Sie ist an der Wahrnehmung von räumlicher Lokalisation und dem „wo“ von
Objekten beteiligt. 2. Verhaltenskontrolle, Dorsalbahn steuert Interaktion mit dem Objekt.
Beschreiben Sie die Lage des primären auditorischen Cortex und nennen Sie 2 wichtige
Organisationsprinzipien!




Brodman-Areal 41,42
Liegt in der Sylvischen Furche
Der PAC ist in funktionellen Säulen organisiert. Alle vertikal untereinander liegenden Neurone
antworten am stärksten auf Töne desselben Frequenzbereichs
Ist tonotop organisiert: weiter anterior gelegene Gebiete des PAC reagieren auf Töne hoher
Frequenz - weiter posterior gelegene Gebiete reagieren auf Töne niedriger Frequenz
Vom Ohr zum primären auditorischen Kortex



Keine „Hörbahn“, sondern Netz von Bahnen
Schall → Schwingung des Trommelfells → Schwingung der Mittelohrknöchelchen → Weiterleitung
ins Innenohr (Cochlea) → Schwingung der Haarzellen → Umwandlung in APs → über Hörnerv zu
oberen Olivenkernen → über tractus lemniscus lateralis zu Colliculi inferiores → corpora
geniculata mediales im Thalamus → primärer aud. Kortex
Signale aus einem Ohr werden sowohl in ipsi- als auch kontralateralen aud. Kortex geleitet
Wie funktioniert die Lokalisation von Geräuschen?




Obere Olivenkerne sind verantwortlich
Ein Geräusch, das von links kommt erreicht zuerst das linke Ohr und umgekehrt
Einige Neurone der medialen oberen Olivenkerne sind für geringe Differenzen in den
Ankunftszeiten auf beiden Ohren empfindlich → interaurale Laufzeitdifferenz
Einige Neurone der lateralen oberen Olivenkerne für geringe Amplitudenunterschiede
Wie ortet die Schleiereule Geräusche in der Horizontalebene?


Ortung der Schallwellen in der horizontalen Ebene durch Vergleich der untersch. Lautstärken in
beiden Ohren
Gesichtsschleier verbessert die Lokalisierung, da er als Reflektor hochfrequenter Schallwellen
fungiert
Wie ortet die Schleiereule Geräusche in der Vertikalebene? (senkrecht zur Oberseite des
Kopfes)

Wichtig hierfür ist der Gesichtsschleier der Eule. Auf der rechten Seite ist er etwas nach oben
gerichtet, weshalb die Eule mit dem rechten Ohr etwas empfindlicher reagiert auf Geräusche
oberhalb der Horizontalebene. Auf der linken Seite ist sie empfindlicher für Geräusche unterhalb
der Horizontalebene, da der Schleier etwas nach unten gezogen ist. Ohne Gesichtsschleier
funktioniert die Ortung nicht (bei komplexen und reinen Tönen)
Wozu führen Schädigungen des auditorischen Kortex?


Keine Beeinträchtigung der Tonwahrnehmung, aber Lokalisationsdefizite
Worttaubheit: Störung des Erkennens von Sprache, ohne dass Fähigkeit zur
Geräuschwahrnehmung verloren geht
Beschreiben Sie die Lage des primären somatosensorischen Kortex im menschlichen Gehirn.
Was versteht man unter einem „somatosensorischen Homunkulus“? 1 Punkt



Liegt auf dem Gyrus postcentralis des Parietallappen
Unter einem somatosensorischen Homunkulus versteht man die somatotope Karte, die den
primären somatosensorischen Cortex umfasst. Der primäre somatosensor. Kortex weist eine
Organisation auf, die einer Karte der Körperoberfläche entspricht.
Größter Bereich des primären somatosens. Kortex erhält Signale aus Körperteilen, die feines
taktiles Unterscheidungsvermögen haben (Hände, Lippen, Zunge)
Wie ist der primäre somatosensorische Kortex organisiert?


Vier parallele Streifen mit ähnlicher, aber gesonderter somatotoper Organisation
Jeder Streifen ist für eine untersch. Art von somatosens. Signalen empfindlich (Druck,
Temperatur) → Neuronen, die horizontal zu einander liegen, antworten bevorzugt auf eine von 4
Arten taktiler Reizung (sind in dem Punkt also untersch.), aber antworten alle auf Reize aus
demselben Teil des Körpers
Nenne zwei Arten von Hautrezeptoren!


Freie Nervenendigungen: reagieren besonders auf Temperatur und Schmerz
Pacini-Körperchen: reagieren auf plötzliche mechanische Belastung der Haut
Nennen Sie drei Systeme der Sensomotorik!



Exterozeptiv: über das externe Reize wahrgenommen werden, die auf Haut einwirken
-mechanische Reize; thermische Reize; nocizeptive Reize (Schmerz)
Propriozeptiv: Analyse der Info über Lage des Körpers (von Muskeln, Gelenken,
Gleichgewichtsorgane)
Enterozeptiv: allg. Info über Zustände innerhalb des Körpers (Temp., Blutdruck,…)
Was ist Stereognosie?

Identifikation von Objekten durch Tasten
Nenne die zwei wichtigsten aufsteigenden somatosensorischen Bahnen!


Lemniscus-Hinterstrangsystem: projeziert direkt in primären somatosensor. Kortex;
Umschaltstellen im Thalamus
Somatosens. Bahn, durch die Tastinfo und Info zur Propriozeption in der dorsalen weißen
Substanz des Rückenmarks zur Medulla aufsteigen
Vorderseitenstrangsystem: diffuse Projektion; Umschaltstellen im Thalamus
Übertragung von nocizeptiver und thermischer Info; in Vorderseitenstrang der weißen Substanz
des Rückenmarks
Wie und wo tritt Info von somatosens. Rezeptoren ins Rückenmark?


Info wird über neuronale Fasern übertragen, die sie sich zu peripheren Nerven vereinigen und
dann über die Hinterwurzel ins Rückenmark eintreten.
Dermatom: der Bereich des Körpers, der durch die beiden Hinterwurzeln eines
Rückenmarksegments innerviert wird
Schädigungen des somatosensorischen Systems




Nur geringe Folgen bei Schädigung des primären somatosens. Kortex
(Agnosie: Unfähigkeit best. sensor. Reize zu erkennen, obwohl keine Störung der sensor.,
verbalen oder geistigen Funktionen vorliegt):
Astereognosie: Unfähigkeit Objekte durch Tasten zu erkennen
Asomatognosie: Unfähigkeit, eigene Körperteile zu erkennen; häufig non Anosognosie begleitet
(Leugnen der Symptome) oder kontralateralem Neglect (nicht auf Reize im kontralateralen
Gesichtsfeld einer rechtshemisphärischen Verletzung reagieren)
Schmerzwahrnehmung unterscheidet sich in mehreren Punkten von anderen
Wahrnehmungssystemen. Man spricht auch von „Paradoxien der Schmerzwahrnehmung“.
Beschreiben Sie diese Paradoxien in Stichworten!



Adaptiver Wert: Schmerz ist zwar unangenehm, aber lebensnotwendig als Schutz →Lernen aus
Schmerzerfahrung
Keine kortikale Schmerzrepräsentation: Es gibt keine Gehirnareale die für die
Schmerzempfindung zuständig ist, aber Gyrus cinguli immer beteiligt (bei Entfernung: gleiche
Empfindungsschwelle, aber Gleichgültigkeit)
Absteigende Schmerzkontrolle: Schmerz kann mit emotionalen und kognitiven Faktoren
unterdrückt werden → Kontrollschrankentheorie: Signale aus Gehirn können über zentrifugale
Bahnen (Info von höherer zu niedriger Ebene)neuronale Schaltkreise im Rückenmark aktivieren,
um Schmerzsignale zu blockieren →Stimulation des periaquaeductalen Grau hat analgetische
Wirkung (Opiate)
Was ist Phantomschmerz?
Wahrnehmung von Schmerzen in amputiertem Körperteil → kortikales Produkt
Was bedeutet Aroma?


Olfaktorisches und gustatorisches System werden als chemische Sinne bezeichnet, da sie chem.
Stoffe der Umgebung registrieren
Beim Essen sind beide Sinne beteiligt →integrierter Wahrnehmungseindruck: Aroma
Was sind Pheromone? Welche Aktivitäten fördern sie?

Pheromone sind Duftstoffe, die der biochemischen Kommunikation zwischen Lebewesen
einer Spezies dienen. Andere Arten bleiben von dieser Kommunikation ausgeschlossen.
Pheromone können das soziale Verhalten von Artgenossen beeinflussen und dienen als
Sexuallockstoffen, zur Wegmarkierung oder als Alarmbotenstoff.
Welche Besonderheit gegenüber anderen Sinnessystemen weist das olfaktorische System
auf?

Sendet seine Signale nicht über Thalamus zur Großhirnrinde (kein primärer sensor. Kortex)
Von der Nase zum Kortex


Olfaktorische Rezeptoren in Riechschleimhaut → Axone durchqueren Siebbeinplatte des
Schädels → Bulbus olfactoris (Riechkolben, erster Hirnnerv) → synapt. Verbindungen zu
Neuronen, die über tractus olfactorius ins Gehirn projizieren (Amygdala und Cortex piriformis)
Zwei wichtige Bahnen verlassen amygdala-piriformes Gebiet: eine projiziert diffus auf limbisches
System (Vermittlung emotionaler Reaktion auf Geruchsreiz); die andere projiziert über Nuclei
medialis dorsalis des Thalamus zum orbitofrontalen Cortex (Vermittlung bewusster
Geruchswahrn.)
Von Zunge zu Kortex


Geschmacksrezeptoren auf Zunge (in Geschmacksknospen gruppiert) → über nervus facialis
(VII), nervus vagus (X) und nervus glossopharyngeus (IX) → Neurone des nucleus solitarius in
medulla oblongata projizieren auf → nucleus ventralis posteriomedialis des Thalamus → Axone
ziehen zu primärem gustatorischem Kortex (sylvische Furche) und dann zum sekundären gust.
Kortex (tief in sylvischer Furche)
Besonderheit: Nervenbahnen verlaufen hauptsächlich ipsilateral
Nenne zwei Folgen von Hirnschäden auf die chemischen Sinne!


Anosmie: Unfähigkeit zur Geruchswahrnehmung, häufig nach Schädel-Hirn-Trauma → Axone
reißen bei starker Erschütterung
Ageusie: Unfähigkeit zur Geschmackswahrn.; sehr selten, da es 3 Projektionsbahnen gibt; bei
Ohrverletzungen, weil nervus facialis durch Mittelohr verläuft
Kapitel 9
Was versteht man unter einem sekundären motorischen Kortex?

Gebiet der Großhirnrinde, das die meisten Eingänge aus dem Assoziationscortex erhält und
selbst hauptsächlich auf den primären motor. Kortex projiziert
Aus welchen zwei Hauptgebieten besteht der sensomotorische Assoziationskortex?




Posteriorer parietaler Assoziationskortex: Integration von sensorischer Info zur effizienten
Ausübung von Handlungen →Empfang von räumlicher Info (aus visuellem, auditorischem und
somatosensorischem System) → um Willkürbew. zu steuern
Projektionen in frontale Cortexgebiete (dorsolateraler präfrontaler Assoziationsk., Areale des sek.
motor. K., frontales Augenfeld (Gebiet des präfrontalen K. zur Kontrolle v. Augenbew.)
Dorsolateraler präfrontaler Assoziationskortex: mentale Repräsentation von Reizen, auf die
der Organismus reagieren wird → Einleitung komplexer Willkürbew.
Projektionen in sek. und primären motor. Kortex und frontales Augenfeld
Welche Auswirkungen haben Schädigungen des posterioren parietalen Assoziationskortex?



Apraxie: Unfähigkeit zu Willkürbewegungen, aber ohne Nachdenken keine Probleme bei
derselben Bew. → Schädigung des linken posterioren Parietallappens → aber bilaterale
Symptome
Konstruktive Apraxie: Unfähigkeit, Bew. auszuführen, um ein Objekt zusammenzusetzen
(Puzzle)
→ Schädigung des rechten posterioren Parietallappens → bilateral
Kontralateraler Neglect: Patient reagiert nicht mehr auf visuelle, auditorische und
somatosensorische Reize auf der zur Läsion kontralateralen Seite (meist links) → Schädigung
meist im rechten Parietallappen
Welche Rolle spielt das Kleinhirn bei Bewegungen?

Erhält Infos aus primärem und sekundärem motor. Kortex, Infos über absteigende Signale
aus motor. Kernen des Hirnstamms und Feedback über motor. Akt. über
somatosensorische Systeme → Vergleich der Eingänge

Korrektur gerade ablaufender Bew.; wichtige Rolle beim Erlernen neuer Bew.abläufe
Welche Rolle spielen die Basalganglien?


Modulatorische Aufgaben; Beteiligung am sensomotorischem Lernen
Bestandteil neuronaler Schleifen, die Eingänge aus versch. Gebieten des Kortex erhalten
und diese über den Thalamus zurück in versch. Gebiete des motor. Kortex schicken
Erläutern Sie das Prinzip der reziproken Innervation am Bsp. des Schutzreflexes. (2 Pkt.)

Prinzip der Verschaltungen im Rückenmark, aufgrund derer ein Muskel automatisch
erschlafft, wenn sich sein Antagonist kontrahiert

Aufgrund eines Schmerzreizes, zum Beispiel an der Hand, feuern die sensomotorischen
Neurone, wodurch die Motoneurone des Bizeps aktiviert und die Motoneurone des Trizeps
gehemmt werden (= reziproke Innervation). Dadurch kommt es zum Beugen des Ellenbogens und
einem automatischen Zurückziehens der Hand (Schutzreflex)
Was versteht man unter Response Chunking? Nennen Sie ein Beispiel.


Response Chunking: Durch Übung werden zentrale sensomotorische Programme, die
Einzelbewegungen kontrollieren, zu Programmen kombiniert, die ganze
Verhaltenssequenzen steuern. Außerdem können einzelne kombinierte Verhaltensweisen
wiederum zu übergeordneten größeren Stücken verbunden werden. Ein Beispiel ist das Tippen
gut gelernter und oft wiederholter Worte, wie beispielsweise Passworte oder die eigene Adresse,
oder oft wiederholte Telefonnummern, die sich fast von alleine wählen.
 Verlagerung auf untere Ebenen um die höheren für z. B. mentale Prozesse frei zu machen
Was ist ein „motorischer Homunculus“?
Ein motorischer Homunkulus (lat. „Menschlein“) ist eine Punkt zu Punkt Repräsentation der
Köperperipherie und dem Gehirn. Diese Projektionen vom Körper auf das Gehirn entsprechen den
motorischen Rindenfeldern. Die somatotope Karte, die den primären motorischen Cortex
umfasst.
Nennen Sie die drei Prinzipien nach denen das sensomotorische System funktioniert.




Das sensomotorische System ist ein parallel aufgebautes, in Funktionseinheiten gegliedertes
hierarchisches System. Sensomotorisches Feedback über Rückkopplungs-Schaltkreise:
Überwachung der Effektivität durch Augen, Gleichgewichtsorgan, Haut-, Muskel-,
Gelenkrezeptoren → dabei gewonnene Info wird an sensomotor. Schaltkreise zurückgesendet
Sensorische Info steuert motorische Aktivität
Lernen verändert sensomotorische Kontrolle
Hauptunterschied zum sensorischen System: Hauptrichtung des Infoflusses: in sensor. Systemen
hauptsächlich die Hierarchie aufwärts; im sensomotor. abwärts
Wo liegt der primäre motorische Cortex? Nenne 2 Organisationsprinzipien!




Gyrus praecentralis des Frontallappens (Brodman-Areal 4)
Der PMC ist somatotop aufgebaut. Ausnahme: Es gibt keine somatotop getrennten
Fingergebiete, sondern die Kontrolle der einzelnen Finger wird vom Hand-Gebiet gesteuert.
Jede Stelle im primären motorischen Cortex kontrolliert die Bewegungen einer bestimmten
Muskelgruppe und erhält Feedback über den somatosensorischen Cortex.
Muskeln in den unteren Körperpartien werden durch Neurone im oberen Teil des
motorischen Projektionsfeldes gesteuert und umgekehrt.
Hauptursprungsort motorischer Signale aus Cortex in untergeordnete Ebenen
Das sensomotorische System ist hierarchisch organisiert und umfasst u.a.
auch sekundäre motorische Areale. Welche dieser Areale kennen Sie? Was ist
ihre Aufgabe?







supplementär-motorisches Areal
(Oberseite des Frontallappens bis in die Fisura longitudinalis)
die meisten direkten sensorischen Eingänge aus somatosensor. System
Ist zuständig für
komplexe, ungeübte und selbstinitiierte Bewegungsverarbeitung und
sequentielle Bewegungen und deren Erlernen
Prämotorischer Cortex:
(Bildet einen Streifen vom supplementär-motorischen Areal bis zur Fissura lateralis)
Die meisten sensor. Eingänge aus visuellem System
Ist zuständig für
extern ausgelöste Bewegungen,
Speicherung, Generierung und Abruf von motorischen Programmen,
Kontrolle der Stütz- und Haltemotorik,
Vorstellung komplexer Bewegungsmuster,
Erkennen und Verstehen von bedeutungstragenden Bewegungen (Mirror Neurons)
Die motorischen Areale des Gyrus cinguli (unmittelbar inferior vom supplementör-motor. Areal)
Wie werden die Neurone aus dem primären motor. Kortex zu den Motoneuronen des
Rückenmarks weitergeleitet?






über 2 dorsolaterale Bahnen und 2 ventromediale Bahnen
dorsolaterale Bahnen: lateraler Corticospinaltract (direkt) und Corticorurospinaltrakt (indirekt)
ventromediale Bahnen: anteriorer Corticospinaltrakt (direkt) und Corticobulbospinaltrakt
(indirekt)
Corticobulbosoinaltrakt: interagiert mit Tectum (audit. U. visuelle Info über räuml. Lage);
Vestibularkern (Gelichgewichtsinfo aus Innenohr); Formatio reticularis (motor. Programme für
komplexe arttyp. Bew.) und motor. Kerne der Hirnnerven, die Gesichtsmuskeln kontrollieren
Direkte Bahnen: Axone dirket zum Rückenmark
Indirekte Bahnen: Axone zu Neuronen im Hirnstamm, die dann Fortsätze ins Rückenmark senden
Welche Funktion haben die 4 absteigenden motorischen Bahnen?


Ventromediale: Kontrolle der Körperhaltung und Bew. des Gesamtoragnismus
Dorsolaterale: Kontrolle von Greif- und Streckbew. ; indirekte außerdem für unabh. Fingerbew.
Was ist ein motorischer Pool?


Gesamtheit aller Motoneurone eines Muskles
Motorische Einheiten: bestehen aus nur einem Motoneuron und alle von ihm innervierten
Skelettmuskelfasern →wenn das Motoneuron feuert, kontrahieren alle Muskelfasern der Einheit
In welche zwei Kategorien lassen sich die meisten Skelettmuskeln einteilen?




Beuger (Flexoren): beugen ein Gelenk (z.B. Bizeps beugt Ellenbogengelenk)
Strecker (Extensoren): strecken ein Gelenk (z.B. Trizeps streckt Ellenbogengelenk)
Synergetische Muskeln: Kontraktion ruft dieselbe Bew. hervor
Antagonistische M.: arbeiten gegeneinander
Welche Arten von Muskelkontraktionen gibt es?



Isometrische: Spannung zw. 2 Knochen wird erhöht, ohne dass Muskel sich verkürzt und die
Knochen zusammenzieht
Isotonische: Spannung weil Muskel sich verkürzt und die Knochen zusammenzieht
Muskelspannung nimmt zu, wenn entweder mehr Motoneurone des motorischen Pools feuern
oder die Feuerfrequenz zunimmt oder beides
Nennen Sie die zwei Rezeptorarten (Muskelrezeptororgane), die die Arbeit des Muskels
überwachen!


Golgi-Sehnenorgane: reagieren auf Anstieg der Muskelspannung → Schutzfunktion: wird
Kontraktion zu stark, sorgen sie für Erschlaffung
Muskelspindeln: reagieren auf Veränderung der Muskellänge; Rückkopplungsschaltkreis
Was ist rekurrente Hemmung (Renshaw-Hemmung)?


Hemmung eines Neurons durch seine eigene Aktivität verursacht, da eine kollaterale Verzweigung
seines Axons mit einem inhibitorischen Interneuron verschaltet ist (Renshaw-Zellen)
Jedes Mal, wenn ein Neuron feuert inhibiert es sich selbst und andere Neurone seines Pools
übernehmen die Aufgabe
Warum ist Gehen ein komplexer sensomotorischer Reflex?
Visuelle, somatosensorische und Gleichgewichtsinfo muss integriert werden → Bewegungsabfolge
erzeugen; Flexibilität, um auf veränderte Bed. zu reagieren
Hauptsächlich von neuronalen Schaltkreisen im Rückenmark kontrolliert
Zusammenfassung:
Posterior parietaler Assoziationskortex → dorsolateraler präfrontaler Assoziationskortex →
supplementär-motorische Areale und prämotorischer Kortex →primärer motor. Kortex →
dorsolaterale und ventromediale absteigende Bahnen des Rückenmarks → spinale sensomotor.
Schaltkreise → Muskeln
Wichtige Rolle des sensor. Feedbacks auf jeder Ebene!!
Was besagt die Theorie der zentralen sensomotorischen Programme?








Alle Stufen des Systems, außer die höchsten Ebenen, haben best. Aktivitätsmuster
einprogrammiert → alle komplexen Bew. entstehen, indem passende Kombinationen aktiviert
werden
Nach Aktivierung kann jede Ebene, ohne direkt Kontrolle einer übergeordneten Struktur, mithilfe
der Rückkopplungsschaltkreise arbeiten → Bewegungen unbewusst
Übergeordnete Strukturen halten sich Kontrolle aber offen
Kontrolle der Effektivität durch Cerebrellum und Basalganglien
Manche Programme stehen ohne explizites Üben zur Verfügung
Andere entstehen durch Übung:
Response chunking
Verlagerung der Kontrollebene auf niedrige Niveaus während des Übens: → höhere Ebenen sind
frei für mentalen Aspekten der Aktivität; Geschwindigkeitssteigerung, da mehr untergeordnete
Schaltkreise gleichzeitig agieren können, ohne sich zu stören
Kapitel 10
Nennen Sie zwei Theorien des Essverhaltens

Anreiztheorie
Menschen und Tiere werden nicht durch ein internes Energiedefizit zum Essen getrieben, sondern
durch die erwarteten angenehmen Effekte der Nahrung zum Essen verlockt. Die Stärke des
Hungergefühls, das jemand empfindet, hängt vom Anreiz ab, den Essen für ihn in diesem
Augenblick hat. Dieser Wert wird von verschiedenen Faktoren beeinflusst: erwarteter Geschmack;
Zeitraum, der seit der letzten Mahlzeit vergangen ist; Tageszeit in Bezug auf die gewöhnlichen
Essenszeiten; Art und Menschen anwesend sind.

Sollwerttheorie
Hunger wird auf ein Energiedefizit zurückgeführt; Essen ist das Mittel, durch das die
Energiereserven des Körpers auf ihren Energiesollwert zurückkehren. Sinkt der Spiegel der
Energiereserven weit genug unter den Sollwert, wird man durch Hunger dazu motiviert, zu essen.
Nach der Sollwerttheorie wird so lange weiter gegessen, bis der Energiespiegel wieder auf seinen
Sollwert zurückgekehrt ist und man sich gesättigt (nicht mehr hungrig) fühlt
Was besagt die Sollwert-Hypothese?





Essen als Mittel, um Energiesollwert wieder herzustellen
Sollwertmechanismus definiert Sollwert
Fühlermechanismus registriert Abweichungen (Blutzuckerspiegel oder Körperfett)
Effektormechanismus eliminiert diese Abweichungen (Hunger → Essen)
Sollwertsysteme sind Systeme mit neg. Feedback: Veränderung in der einen Richtung ruft
kompensatorische Effekte in die andere Richtung hervor → Aufrechterhaltung der Homöostase
Welche zwei Sollwertsysteme und Regelgrößen wurden angenommen?
Glucostatische Theorien: Kohlenhydrate → Hungergefühl entsteht durch Absinken des
Blutzuckerspiegels unter Sollwert
Lipostatische Theorien: Fettgehalt des Körpers → Hungergefühl durch Absinken des Fettgehaltes
unter Sollwert
Wechselspiel dieser beiden Systeme: glucostatisches Kurzzeitsystem und lipostatisches
Langzeitsystem
Was spricht gegen Sollwerttheorien?



Evolutionstheorie: in unsicheren Zeiten beugt man Energiedefiziten vor, indem man soviel ist wie
möglich → nicht nur Reaktion auf Energiedefizite
Sollwertregulation konnte im Exp. nur bei extremen Abweichungen vom Sollwert festgestellt
werden → Versuchstieren wurde unnatürlich viel Insulin injiziert → auf kleinere, natürl.
Abweichungen reagiert Körper nicht
Nicht-Berücksichtigung wichtiger Faktoren wie Geschmack, Lernen und soz. Einflüsse (z.B.
Exp. zum Scheinessen)
Nennen Sie drei Phasen des Energiestoffwechsels!



Cephalische Phase: Vorbereitungsphase; eingeleitet durch Erwartung von Nahrungsaufnahme
→ Bauchspeicheldrüse schüttet große Mengen an Insulin aus und nur wenig Glucagon, um
Glukosespiegel zu senken
Resorptive Phase: Nahrungsaufnahme: aktueller Energiebedarf wird gedeckt, Überschuß
gespeichert → viel Insulin und wenig Glucagon, um Anstieg der Nährstoffkonzentration im Blut zu
minimieren, indem es Verbrauch und Speicherung fördert
Fastenphase: Zeit zw. Nahrungsaufnahmen: Energie ist verbraucht, um Energiebedarf zu decken
muss auf gespeicherte Energieressourcen zurückgegriffen werden → wenig Insulin, viel Glucagon
Was fördert und hemmt ein hoher Insulin- und ein niedriger Glucagonspiegel?


Fördert: Verwertung von Blutzucker als Energiequelle; Umwandlung von Glukose in Glycogen und
Fett; Umwandlung von Aminosäuren in Proteine; Speicherung von Glycogen in Leber und
Muskulatur, von Fett in Fettgewebe und von Eiweiß in Muskulatur
Hemmt: Umwandlung von Glycogen, Fett und Proteinen in direkt nutzbare Energiequellen (wie
Glucose)
Was fördert und hemmt ein niedriger Insulin- und hoher Glucagonspiegel?


Fördert: Umwandlung von Fetten in freie Fettsäuren und deren Verwertung als Energiequelle;
Umwandlung von Glycogen in Glucose, von freien Fettsäuren in Ketone und von Proteinen in
Glucose
Hemmt: Verwertung von Glucose als Energiequelle des Körpers, aber nicht des Gehirns (→
Evolutionsvorteil, wenn Fastenphasen lang → bessere Hirnentwicklung bei ständiger
Nahrungssuche); Umwandlung von Glucose und Fett in Glycogen und von Aminosäuren in
Proteine; Speicherung von Fett in Fettgewebe
Welche Faktoren entscheiden darüber, was wir essen?




Artspezifische Präferenzen (süß, fettig, salzig → Hinweis auf Kalorien und Natrium; Abneigung
gegen bitter → Hinweis auf Giftstoffe)
Erlernte Geschmackspräferenzen und Aversionen: kulturspez. und individuell:
Mögen von Nahrung, auf die Sättigungsgefühl folgt, Abneigung, wenn üble Folgen
Ernährungsmangel: Natriummangel → Vorliebe für Geschmack von Kochsalz;
Vitamin- und Mineralstoffmangel → Tiere entwickeln Abneigung gegen vitaminarmes und lernen,
vitaminreiches zu essen, indem sie die pos. Auswirkungen erleben (denn Vitamine und Mineralien
sind geschmacklos)
Problem heutzutage: künstliche Aromen, ohne entsprechende Nährstoffe; Überangebot an
Nahrungsmitteln
Welche Faktoren entscheiden darüber, wann wir essen?


Hunger vor Mahlzeit wird durch Erwartung der Mahlzeit ausgelöst: Konsum reichhaltiger
Nahrung führt zu einer Störung der Homöostase → Körper versucht diese aufrechtzuerhalten,
indem er Insulin bei den ersten Anzeichen zur Nahrungsaufnahme ausschüttet →
Blutzuckerspiegel sinkt
Klassische Konditionierung des Hungers: Mittagspause → Hunger
Welche Faktoren entscheiden darüber, wie viel wir essen?





Sättigungssignale: abh. von Energiedichte der Nahrung → essen Ratten immer dasselbe
stabilisiert sich bald ihr Essverhalten → gibt man dann neue Kost mit veränderter Energiedichte,
passen sie die Aufnahmemenge an → allerdings klappt das nicht mehr, wenn Energiedicht um
mehr als 50% reduziert wird → außerdem, wenn Schmackhaftigkeit stark verändert wird
Exp. zum Scheinessen bei Ratten: Nahrungsmenge bestimmt durch Erfahrung mit der Wirkung
und nicht durch direkten Effekt (bei bekannter Kost vergrößerte sich die Nahrungsaufnahme erst
nach mehreren Durchgängen, bei unbekannter Kost nahmen Versuchstiere von Anfang an vie zu
sich)
Appetithäppchen-Effekt: kleine Nahrungsmengen vergrößern Hunger, da man über cephalische
Phase nicht hinaus kommt
Essen in Gesellschaft: häufig gesteigerte Nahrungsaufnahme
Sensorisch-spezifische Sättigung: nach Genuss eines best. Nahrungsmittels nimmt Anreiz,
dieses nochmals zu essen ab und Anreiz, etwas anderes zu essen zu
Vorteile: fördert abwechslungsreiche Kost; vielfältiges Angebot fördert Nahrungsaufnahme →
Tiere: Nahrungsüberschuss ausnutzen
Welche Rolle spielt diätinduzierte Thermogenese bei der Entwicklung von Fettleibigkeit? (2
Pkt.)




Diätinduzierte Thermogenese ist der Mechanismus, durch den der Körper die Effizienz seiner
Nahrungsverwertung im Verhältnis zu seinem Körperfettanteil über die Änderung der
Körpertemp. anpasst. Eine Zunahme des Körperfetts führt zu einer Abnahme der Effizienz der
Energieverwaltung → höhere Körpertemperatur (Energie verprassen). Dies wiederum führt dazu,
dass zusätzliche Energie benötigt wird, um den Körperfettanteil aufrechtzuerhalten.
Bei Gewichtsverlust werden Energiequellen effizienter genutzt, um weitern Verlust
entgegenzuwirken
Dicke schwitzen mehr, Dünne frieren eher (→ Energie sparen)
Individuelle Unterschiede im Grundumsatz und der Fähigkeit, Energieumsatz anzupassen
Nenne zwei Modelle zur Regulation des Körpergewichts!



Sollwerttheorien: setzen aktiven Mechanismus voraus, der die Rückkehr des Körperfettniveaus
zu einem festen Bezugspunkt sichert
→ Problem: Variabilität des Körpergewicht auch im Erwachsenenalter; pos. Effekt von
Nahrungsreduktion: Tiere, deren Nahrungsaufnahme beschränkt wurde, wurden schlanker, lebten
gesünder und länger; Körpergewicht nicht nur Funktion der Nahrungsmenge, sondern Fettanteil
wird auch durch Anpassung der Effizienz der Nahrungsverwertung kontrolliert (→diätinduzierte
Thermogenese)
Bezugspunktmodelle: Körpergewicht schwankt um einen natürl. Bezugspunkt (Niveau bei dem
die gewichtsfördenden und –senkenden Faktoren in Gleichgewicht sind) → homöostatische
Regulation ohne aktiven Rückkehrmechanismus zum Sollwert → solange es keine langfristigen
Veränderungen gibt, bleibt Körpergewicht konstant → bei langfristigen Veränderungen ändert sich
Bezugspunkt → Feedback-System limitiert nur weitere Veränderungen in derselben Richtung
(nicht zurück zum Sollwert)
Nennen Sie vier Befunde zur Gewichtregulierung!





Gesamtanteil des Körperfetts normalerweise konstant
Bei vielen verändert sich das Gewicht häufig (→spricht gegen Sollwert)
Stoffwechseländerungen nach Reduktion/Erhöhung der Nahrungsaufnahme, um einer
weiteren Veränderung entgegenzuwirken, nicht um auf Sollwert zurückzukehren
Nach Gewichtsverlust besteht Tendenz, das urpsrüngl. Gewicht wiederherzustellen
→ um Gewicht dauerhaft zu verändern, müssen Essgewohnheiten dauerhaft geändert werden!!!
Störungen






Diabetes mellitus: Störung des Fettstoffwechsels, vermehrter Körpereiweißabbau → hohes
Ausscheiden von Blutzucker, Verringerung der Glukoseaufnahme
Erbrechen: evolutionsbiologischer Schutzreflex, um Aufnahme giftiger Substanzen zu verhindern
Störungen des Essverhaltens: (dürfte es laut Sollwerttheorie nicht geben)
Fettleibigkeit (Adipositas): viel Essen hat adaptiven Wert, aber in unserer heutigen Zeit nicht
mehr nötig; Missverhältnis zw. hoher Energieaufnahme und niedrigem Verbrauch;
Rückkopplungssignale (Sättigungssignal): Insulin und Leptin
Magersucht (Anorexia nervosa): krankhafte Furcht vor Übergewicht, die zu extremen
Gewichtsverlust führt → Nahrungsverweigerung
Bulima nervosa: Fressattacken → Erbrechen, Abführmittel, …
Kapitel 11
Wie kann perinatales (Zeit um die Geburt) Testosteron das Gehirn maskulinisieren?


Tierversuche haben ergeben, dass Testosteron das Gehirn nicht direkt maskulinisiert, sondern die
Maskulinisierung geschieht durch Östradiol. Östradiol entsteht bei der Aromatisierung von
Testosteron: chem. Prozess, bei dem ein Cyclohexanring in einen Benzolring umgewandelt wird.
Weibliche Feten werden durch die plazentare Schranke vor den maskulinisierenden Effekten des
mütterlichen Östradiols geschützt (durch Alphafetoprotein, das an Östradiol bindet (Ratten)).
Welche Hormone schüttet der Hypophysenvorderlappen aus? Welche Funktionen haben sie in
der Sexualentwicklung?


Der Hypophysenvorderlappen schüttet v.a. glandotrope Hormone aus. Das sind Hormone, die
die Hormonproduktion anderer endokriner Organe regulieren. Solche glandotrope Hormone sind
unter anderem die Gonadotropine, die die Ausschüttung der Keimdrüsenhormone stimulieren,
und das adrenocorticotrope Hormon (ACTH), welches die Hormonausschüttung der
Nebennierenrinde stimuliert.
Während der Pubertät kommt es zur vermehrten Ausschüttung von Gonadotropinen und ACTH,
was zur Ausbildung der sekundären Geschlechtsmerkmale und zur Reifung der Genitalien
führt. Außerdem schüttet der HVL Wachstumshormone (nicht glandotrop) aus, was den
pubertären Wachstumschub zur Folge hat
Diskutieren Sie am Beispiel der „testikulären Feminisierung“ die wichtige Rolle der Androgene
(insbesondere der Testosterone) für die menschliche Sexualentwicklung.








Testikuläre Feminisierung = Genotyp ist männlich (XY, keine Eierstöcke, innenliegende Hoden,
männlicher Hormonspiegel), Phänotyp ist weiblich. Grund ist eine AndrogenUnempfindlichkeit, d.h., der Körper reagiert nicht auf Androgene (z. B. Testosteron).
Aufgrund des XY-Chromosoms bilden sich aus den Medullen der Primordialgonaden Hoden
Im 3. Monat wird in der männlichen Fetalentwicklung von den Hoden Testosteron und das
Müllersche inhibierende Hormon ausgeschüttet
Testosteron führt normalerweise zur Entwicklung der Wolffschen Gänge zu inneren männlichen
Geschlechtsorganen → aufgrund der Unempfindlichkeit gegenüber Androgenen, entwickeln sie
sich nicht
MHH unterdrückt trotzdem die Entwicklung der Müllerschen Gänge zu weiblichen
Geschlechtsorganen
Das bipotente Vorläuferorgan, aus denen sich die Genitalien entwickeln wird auch von der Anbzw. Abwesenheit von Testosteron gesteuert → es entwickeln sich weibliche Genitalien; Hoden
bleiben im Körper
Gehirn und Verhalten bilden sich nach vorprogrammiertem weiblichem Schema, da
Androgenunempfindlichkeit zu keiner Unterdrückung führt
In Pubertät produzieren die Hoden genügend Östrogen, um Körper zu verweiblichen, da
Androgen nicht entgegenwirken können
Kapitel 12
Nennen Sie die Schlafstadien sowie deren typische EEG-Muster !



Wachzustand: desynchronisiertes EEG: geringe Amplitude, hohe Frequenz
Vor dem Einschlafen zunehmende Alpha Wellen (Zu & abnehmende Ausschläge von 8-12 Hz)
Schlafstadium 1
hochfrequentes Signal mit niedriger Amplitude, mit EEG im Wachzustand
ähnlich, jedoch langsamer. → Initialstadium: keine EOG/EMGVeränderungen (orthodox); alle weitern Perioden: REM und Verlust des
Muskeltonus (paradox)
 Schlafstadium 2
zwei typische Signalformen treten auf: K-Komplexe (ein+ und einAusschlag) und Schlafspindeln (1 bis 2 Sekunden, 12-14 Hz)
 Schlafstadium 3
gelegentliches Auftreten von Delta-Wellen
 Schlafstadium 4
vollständig von Delta-Wellen dominiert
 Im Verlauf des Schlafs: sukzessive Zunahme der Amplituden und Abnahme der Frequenz
 2,3,4: Slow-wave-Sleep (SWS); Stadium 1: REM-Schlaf
 Schlafzyklus: Durchlauf von 1 bis 4 und wieder zurück; 90 min.; REM-Perioden immer länger
Nennen und beschreiben Sie kurz 2 Theorien des Schlafes!


Restaurative Theorie:
destabilisierende Auswirkungen des Wachseins werden durch Schlaf zu Recht gerückt
Circadiane Theorie:
Schlafsystem von biorhythmischer Uhr gesteuert um Energie zu sparen und als Schutz vor
Gefahren der Umwelt → Bedürfnis nach Schlaf, um Energievorräte zu schonen und sich nicht
Gefahren auszusetzen
Was besagen circadiane Schlaftheorien? Was spricht für und was gegen circadiane ST?



Schlaf ist in der Evolution als neuronaler Mechanismus entstanden, der Tiere in Zeiten, in denen
sie nicht damit beschäftigt sind ihr Überleben zu sichern, inaktiv werden lässt. Das Bedürfnis nach
nächtlichem Schlaf entwickelte sich beim Menschen, damit sie Energievorräte schonen konnten
und damit sie während der Nacht keinen Gefahren ausgesetzt waren.
Dafür spricht:
große Unterschiede zw. der Schlafdauer versch. Arten → abh. wie angreifbar während des
Schlafs und wie viel Zeit für Fressen gebraucht wird; nicht etwa weil sie untersch. aktiv sind
dass nach langen Wachphasen kürzere Schlafphasen folgen;
Schlafentzugsexp. : kaum bedeutsame physiologische Veränderungen, nur in Tests, die
Aufmerksamkeit erfordern, neg. beeinträchtigt → Unfähigkeit sich wach zu halten, nicht durch
restaurativ bed. Funktionsabfall während des Wachseins; Microsleeps → Gehirn kompensiert
Schlafentzug mit Erhöhung der Effizienz (Dauer und Qualität des Slow wave sleep), nicht mit
Verlängerung des Schlafs
Bei Kurzschläfern weniger Schlafphase 1 und 2, gleichviel 3 und 4
Dagegen spricht,
Was besagen restaurative Schlaftheorien? Was spricht für und was gegen restaurative ST?

Die restaurative Schlaftheorie besagt, dass der Wachzustand die Homöostase des Körpers stört,
Schlaf dient dazu, sie wiederherzustellen.


Dafür spricht:
Dagegen spricht:
dass Arten, die mehr Energie verbrauchen, länger schlafen müssten. Es gibt jedoch keine
Korrelation zwischen der Schlafdauer einer Art und ihrem Aktivitätsniveau, ihrer
Körpergröße oder der Körpertemperatur.
Nach langen Wachphasen folgen kürzere Schlafphasen. Das spricht für die circadiane
Schlaftheorie und gegen die restaurative, nach der auf lange Wachphasen lange Schlafphasen
folgen müssten → kein Kompensationsschlaf
Was sind freilaufende circadiane Schlafzyklen?





Circadian: dauert einen Tag
Viele Arten haben einen regelmäßigen Tag-Nacht-Rhythmus → gesteuert durch Zeitgeber
(Hinweisreize aus Umwelt, v.a. Hell-Dunkel; konditionierbar)
Freilaufende circadiane Schlaf-Wach-Zyklen bleiben auch ohne Zeitgeber stabil: freilaufende
Perioden konstant und dauern ca. 25h (innere biologische Uhr: nach 25h schläft ein Mensch in der
Regel ein) → müssen nicht gelernt werden
Auch hier ist die Korrelation zw. Dauer der Wachperiode und anschließender Schlafperiode
negativ
Andere freilaufende circadiane Rhythmen: Körpertemperaturzykus: geht mit Schalf-WachZyklus einher → Schlaf während Temperatur abnimmt; Erwachen bei Temp.zunahme → bei
konstanten Laborbedingungen driften die Zyklen auseinander (Desynchronisation) → spricht für
mehrere circadiane Zeitgeber
Was bewirkt REM-Schlafentzug?



Kennzeichen des REM-Schlafs: schnelle Augenbew., Verlust des Muskeltonus; EEG-Merkmale
des Schlafstadiums 1; eventuell Traumphase
Tendenz mit REM-Phasen zu beginnen steigt; in den ersten drei Nächten nach Schlafentzug ist
der Anteil des REM-Schlafs größer als sonst
Kompensatorische Zunahme des REM-Schlafs → Dauer unabh. von Dauer des SWS geregelt →
andere Funktionen: Erhaltung der geistig-seelischen Gesundheit; Erhaltung des normalen
Antriebniveaus; Verarbeitung von Gedächtnisinhalten (keine Theorie uneingeschränkt bestätigt)
Integration von restaurativen und circadianen Schlaftheorien

Effekte des circadianen Faktors (biologische Uhr) und Effekte des restaurativen Faktors
(Wachsein fördert Schläfrigkeit) können sich überlagern (Borbély)
An der Steuerung des Schlafverhaltens sind eine Reihe neuronaler Strukturen beteiligt. Welche
Rolle spielt die „Formatio Reticularis“ bei der Schlafsteuerung? Nennen Sie Beispiele. 1 Punkt


REM-Schlaf wird von Vielzahl von Hirnregionen gesteuert, die über die Formatio reticularis
verstreut liegen; jede Region ist für anderes Symptom des REM-Schlafs verantwortlich (Abnahme
des Muskeltonus, EEG Desynchronisation, schnelle Augenbewegung usw.).
Der zyklisch auftretende REM-Schlaf wird von der Formatio reticularis gesteuert, und zwar durch
eine reziproke Wechselwirkung zwischen exzitatorischen cholinergen (Aktivierung der REMSchlafzentren) und inhibitorischen noradrenergen Neurone des Locus coeruleus sowie
serotonergen Neuronen der dorsalen Raphé-Kerne (Hemmung des REM-Schlafs →
Wiedereinsetzen des SWS).
Was besagt die Schlaftheorie des retikulären Aktivierungssystems?


Schlaf wird aktiv durch Arousalmechanismus in der Formatio reticularis reguliert → reticuläres
Aktivierungssystem (RAS)
Problem: Annahme, dass Schlaf von einer generellen Aktivitätsminderung im RAS hervorgerufen
wird ist nicht vereinbar mit Komplexität der Schaltkreise in der Formatio reticularis
Was besagt die passive Schlaftheorie?

Schlaf als passive Konsequenz der Abnahme des sensorischen Inputs ins Vorderhirn
Nennen Sie drei Befunde zur neuronalen Grundlage des Schlafes!



Kein Zustand neuronaler Ruhe (im REM viele Neurone aktiver als im entspannten Wachzustand)
Schlaffördernde Schaltkreise in Pons und medulla oblongata
Schlafstadien nicht klar trennbar
Welche Gehirnstrukturen werden mit Schlaf in Verbindung gebracht?




Formatio reticularis: Raphé-Kerne, basales Vorderhirn; REM-Schaltkreise
Raphé-Kerne: Ansammlung Serotonin produzierender Kerne; Parachlorphenylalanin (PCPA)
blockiert Synthese von Serotonin und führt bei Katzen zu Insomnie → allerdings haben
Serotonininjektionen nur bei Katzen schlafinduzierende Wirkung
Basales Vorderhirn: Hinweise auf Beteiligung → bilaterale Läsionen führen zu deutlicher
Abnahme der täglichen Schlafdauer bei Katzen; Reizung führt allerdings nicht zu Schlafinduktion
REM-Schaltkreise: siehe oben
Neuronale und molekulare Mechanismen der circadianen Uhr





Freilaufende Schlaf-Wach-Zyklen deuten daraufhin, dass physiologische Systeme der
Schlafregulierung auch durch inneren Zeitgeber gesteuert werden → circadiane Uhr
Nucleus suprachiasmaticus: im Hypothalamus; steuert Fress-, Trink-, Aktivitätsrhythmen; aber
er ist nicht die einzige circadiane Uhr → bei Läsionen bleiben einige circadiane Rhythmen
unberührt; nicht alle Umweltreize, die circadiane Rhythmen steuern werden durch Läsionen
ausgeschaltet
Triggerung der circadianen Uhr: Tag-Nacht-Rhythmus triggert Schlaf-Wach-Rhythmus: Info
über Licht und Dunkel vom Auge → über opt. Bahnen → chiasma opticum → zweigen als Axone
vom Sehnerv ab → projizieren als retinohypothalamische Bahnen zum jeweiligen Nucleus
suprachiasmaticus
Rolle der Epiphyse unklar: bei Vögeln, Amphibien, Reptilien und Fischen wichtige Rolle für
circadiane Rhythmen über Ausschüttung von Melatonin; beim Menschen und anderen Säugern
allerdings haben Läsionen der Epiphyse keine Auswirkungen
Genetische Grundlagen: Fruchtfliege: Gen Per steuert Rhythmen; Hamster: Mutation des Gens
Tau verkürzt Zyklus
Was sind Hypnotika? Nenne ein Beispiel!




Pharmaka, die den Schlaf fördern
Benzodiazepine: fördern kurzfristige Schläfrigkeit; verkürzen Einschlafzeit; weniger Aufwachen;
verlängern Schlafdauer (GABA-Agonist → verstärkt hemmende Wirkung von GABA)
→ Behandlung sporadischer Schlafstörungen
Längere Einnahme: Toleranz gegenüber schlaffördernder Wirkung; Absetzen verursacht
Insomnie; psychische Abhängigkeit; Störung des normalen Schlafmusters
Was sind Antihypnotika? Nenne zwei Hauptklassen!





Pharmaka, die Schlaf hemmen; unterdrücken REM-Schlaf, ohne Gesamtschlafdauer zu
reduzieren
Stimulanzien wie Kokain oder Amphetamine
Trizyklische Antidepressiva
Beide steigern Aktivität der Katecholamine Noradrenalin, Adrenalin und Dopamin durch erhöhte
Ausschüttung bzw. Blockieren der Wiederaufnahme in Synapse
Machen süchtig, starke Nebenwirkungen, Störung des normalen Schlafs
Melatonin

Endogenes Melatoninniveau während Dunkelheit erhöht → exogenes Melatonin kann geringen
Einfluss auf circadiane Rhythmen haben (bei Flügen; bei Insomnien; bei Blinden)
Nenne drei Kategorien von Schlafstörungen!




Insomnie:
Ursachen: iatrogener Ursprung: Beendigung der Einnahme von Schalftabletten; Schalfapnoe
(Atemstillstand in Nacht); nächtliche Myoklonie (Schüttelkrämpfe der Beine); ruhelose Beine
Hypersomnie:
Ursachen: Folge unerkanter Schlafstörungen; Narkolepsie (Behandlung mit Stimulanzien)

REM-Schlafstörungen: Narkoleptiker fallen direkt in REM-Schlaf; Kataplexie: plötzlicher
Muskeltonusverlust im Wachzustand → Zellen des nucleus magnocellularis (Formatio reticularis)
steuern Entspannung (Behandlung mit trizyklischen Antidepressiva); Schlaflähmung: beim
Einschlafen oder Aufwachen Lähmungserscheinungen (REM-Schlaf dringt in Wachzustand)
Was sind polyphasische und monophasische Schlafzyklen?


Polyphasisch: regelmäßig öfter als einmal am Tag (Babys)
Monophasisch: nur einmal innerhalb 24h (Erwachsene)
Kapitel 13
Was macht Kokain zu einer psychoaktiven Substanz?


Psychoaktive Substanzen beeinflussen das NS und wirken sich dadurch auf Erleben und
Verhalten aus.
Kokain steigert die Dopamin- und Noradrenalinaktivität, indem es die Wiederaufnahme aus dem
synaptischen Spalt in den präsynaptischen Endknopf blockiert. Einmal in den synaptischen Spalt
freigesetzte Dopamin- und Noradrenalinmoleküle führen dann zu einer Daueraktivierung der
postsynaptischen Rezeptoren, was zu Euphorie führt.
Was ist Atropin?

Tollkirschengift, Rezeptorblocker mit antagonistische Wirkung, indem es sich an muscarinartigen
Acetylcholinrezeptor bindet und dadurch die Wirkung des Acetylcholins auf diesen Rezeptor
blockiert.
Was sind Stimulanzien? Nennen Sie drei Beispiele




Stimulanzien sind Wirkstoffe, die die neuronale Aktivität und den Betätigungsdrang steigern.
Kokain
Amphetamine
Koffein
Nennen Sie die drei Phasen des Alkoholentzugs und beschreiben sie diese.



1. Phase, 5 bis 6 Stunden nach Alkoholaufgabe. Starke Tremorerscheinungen, Unruhe, Schwitzen
Übelkeit, u.U. Halluzinationen.
2. Phase, 15 bis 30 Stunden nach Alkoholaufgabe. Krämpfe
3. Phase (Delirium tremens), ein bis zwei Tage nach Alkoholaufgabe. Halluzinationen,
Hyperthermie, Herzrasen, Verwirrtheit. Kann tödlich sein
Wie können chemische Substanzen wirken? Erläutern Sie dies an Beispielen!






Chemische Substanzen, im Sinne von Drogen, können auf vielfältige Wiese wirken:
Beeinflussen Synthese, Transport, Ausschüttung und Deaktivierung von NTs
Beeinträchtigung der chem. Reaktionskette in postsynapt. Neuronen:
Agonisten erleichtern die Aktivierung der Synapsen eines Neurotransmitters, indem sie direkt auf
den Rezeptor wirken oder die Synthese, den Abbau und die Wiederaufnahme beeinflussen:
Kokain verhindert die Wiederaufnahme von Dopamin und Noradrenalin in die Synapse,
→Daueraktivierung; Benzodiazepine binden an Untertyp des GABA-Rezeptors und verstärken so
die Bindung von GABA-Molekülen an Rezeptor (Steigerung der inhibitorischen Wirkung)
Antagonisten hemmen die Aktivierung der synapt. Rezeptoren, indem sie an sie binden und sie
so blockieren: Atropin bindet an muscarinartigen ACh-Rezeptor; Curare an nicotinartigen AChRezeptor
Wirkung auf neuronale Membranen des ganzen ZNS → Alkohol
Was versteht man unter kontingenter Wirkstofftoleranz und nenne ein Beispiel für den
Nachweis!

Kontingente Wirkstofftoleranz = Toleranzform, die sich nur gegenüber Drogenwirkungen
entwickelt, die direkt mit einer bestimmten Erfahrung verknüpft ist.

Wird mit Vorher-Nachher-Experimenten untersucht. Ein Beispiel:
In einer Untersuchung der kontingenten Toleranz der krampflösenden Wirkung von Alkohol
wurden zwei Gruppen von Ratten über mehrere Tage Alkohol injiziert. Eine Gruppe erhielt ihren
Injektion eine Stunde vor einer krampfauslösenden Reizung der Amygdala, die andere Gruppe ein
Stunde danach. So konnte die eine Gruppe die krampflösende Wirkung erfahren, die andere nicht.
Am Ende des Versuchs erhielten beide Gruppen ihre Dosis vor der Amygdalareizung. Ergebnis
war, dass die Ratten die immer vor der Reizung die Alkoholinjektion bekamen, eine Toleranz
gegenüber der krampflösenden Wirkung entwickelt haben.
Was sind konditionierte Entzugserscheinungen? Stellen Sie dazu ein Experiment dar!

Konditionierte Entzugserscheinungen sind Entzugserscheinungen die durch die
drogenspezifische Situation oder andere Schlüsselreize, die mit der Verabreichung
verbunden sind erzeugt werden

3 Gruppen Ratten. Eine bekam Morphiuminjektionen im eigenen Käfig und Kochsalzinjektionen in
einer Testumgebung. Bei der zweiten Gruppe war es andersrum. Die dritte Gruppe bekam in
beiden Umgebungen Kochsalz injiziert. Nach jeweils 20 Injektionen pro Tier wurden in der
Testumgebung die die Entzugserscheinungen untersucht. Es zeigte sich, dass die Ratten die
immer in der Testumgebung Morphium bekamen die größten Entzugserscheinungen zeigten.
Was versteht man unter Konditionierter Toleranz? Nenne ein Beispiel für den Nachweis!

Konditionierte Toleranz bedeutet, dass sich die Toleranz nur dann maximal entwickeln kann, wenn
die Droge immer in der gleichen Situation verabreicht wird. Z. B. werden Drogensüchtige tolerant
gegenüber der Droge, wenn sie diese immer in derselben Umgebung einnehmen, was dazu führt,
dass sie immer größere Dosen einnehmen. Nimmt der Süchtige dieselbe Menge allerdings in
einer ungewohnten Umgebung ein, ist die Toleranzwirkung nicht mehr vorhanden, somit nimmt
die Gefahr an einer Überdosis zu sterben zu.

Zwei Gruppen von Ratten bekamen abwechselnd je 20 Alkoholinjektionen und 20 mit
Kochsalzlösung (eine pro Tag). Eine Gruppe bekam die Alkoholinjektion in einem speziellen
Testraum, die Kochsalzlösung in einem Gemeinschaftsraum. Bei der zweiten Gruppe ist es
umgekehrt. Danach wurde die Toleranz auf die hypothermische Wirkung des Alkohols in beiden
Situationen untersucht. Toleranz entwickelte sich nur, wenn die Testinjektion in der Umgebung
verabreicht wurde in der auch vorher Alkohol injiziert wurde.
Was besagt die Theorie der konditionierten Kompensationsreaktion?

Nach der Theorie der konditionierten Kompensationsreaktion entsteht die Wirkstofftoleranz durch
eine pawlowsche Konditionierung: Auf die Drogeneinnahme (UCS) erfolgt normalerweise eine
Kompensationsreaktion (UCR). Wenn nun die Umweltreize (CS) häufiger mit der
Drogeneinnahme zusammen auftreten, reichen diese aus, um die Kompensationsreaktion (CR)
hervorzurufen, da sie die Drogeneinnahme ankündigen.
Nenne zwei biopsychologische Theorien der Abhängigkeit

Theorien der physischen Abhängigkeit
Diese Theorie besagt, dass der Hauptgrund für die Drogensucht eine Vermeidung sowie
Beendigung der Entzugserscheinungen (konditionierte oder reale) ist.

Theorien der positiven Verstärkersysteme
Diese Theorie besagt, dass die angenehmen Wirkungen der Hauptgrund für das Einnehmen von
Drogen sind.
Welche Hirnregion ist bei „intrakranieller Selbstreizung“ betroffen?

Bei der intrakraniellen Selbstreizung ist das mesotelencephale Dopaminsystem betroffen:
System dopaminerger Neurone, die sich vom Mesencephalon in verschiedene Teile des
Telencephalon erstreckt, die Zellkörper liegen in der substantia nigra und dem ventralen
Tegmentum, Endpunkt ist Nucleus accumbens.
Kapitel 14
Welche Auswirkung hat eine selektive Hippocampus-Läsion auf das menschliche Gedächtnis?

Die Konsolidierung des Langzeitgedächtnisses für räumliche Beziehungen wird stark
beeinträchtigt. Neue Umgebungen können nicht mehr gemerkt werden. Das Gedächtnis für
Umgebungen die lange vor der Läsion gelernt wurden, ist nicht betroffen.
Welchen Einfluß haben Läsionen des Riechhirns auf die Leistung in verzögerten
Vergleichsaufgaben bei Ratten?

Läsionen des Riechhirns (Rhinencephalon) erzeugen selbst bei kürzesten
Verzögerungsintervallen Gedächtnisdefizite. Diese Befunde zeigen, dass nicht wie angenommen
der Hippocampus, sondern das Riechhirn die größere Rolle im Objekterkennungsgedächtnis
spielt.
Nennen Sie wichtige Symptome vom Korsakow-Syndrom. Welche Regionen sind betroffen?









Neuropsychologische Störung. Häufig bei Alkoholabusus, aber auch bei Vitaminmangelzuständen
oder Schädelhirntraumata.
anterograde Amnesie für explizite Gedächtnisinhalte: Dabei sind Betroffene nicht in der Lage,
neue Inhalte zu speichern oder wiederzugeben. Aber fast normales implizites Gedächtnis (Test
mit repetition-priming)
retrograde Amnesie: Betroffene können erlebte Inhalte aus der eigenen Vergangenheit nicht
verarbeiten, erkennen oder wiedergeben.
Demenz
Sensorische und motorische Störungen
Persönlichkeitsveränderung
Leber-,Magen-,Darm-,Herzschäden
Läsionen des medialen Diencephalon (v.a. mediodorsale Nuclei des Thalamus; und
Hypothalamus) und vereinzelt des Neocortex und Cerebrellum
Teilweise diffuse Schäden im präfrontalen Cortex
Unter welcher Erkrankung litt N.A.?



Mediale diencephale Amnesie
Nach Verletzung mit Florett → Amnesie →Läsion des linken mediodorsalen Nucleus des
Thalamus, ausgedehnte Läsionen im medialen Diencephalon
→ Beleg für amnestische Folgen einer Läsion des Diencephalons
Welche Gedächtnisstörungen treten bei Läsionen des präfrontalen Kortex auf?



Probleme mit Gedächtnis für zeitliche Abläufe:
Schwierigkeit, sich die Abfolge versch. Ereignisse zu merken (können schon gesehenes aber
wieder erkennen)
Schwierigkeit, Tätigkeiten auszuführen, die eine best. Abfolge von Handlungen erfordern (z.B.
Anziehen)
Nenne Symptome der Alzheimer-Erkrankung! Welche Gehirnregionen sind betroffen?




Neuronale Degeneration, Neurofibrillen und Amyloidplaques im temporalen, frontalem und
parietalem Kortex und cholinerger Neurone des basalen Vorderhirns
Schleichender Verfall bis Demenz (Verlust motorischer und intellektueller Fähigkeiten)
Anterograde und retrograde Amnesie
Störung des Kurzzeitgedächtnisses und bei einigen Formen des impliziten Ged.
Welche Folgen für das Gedächtnis haben Gehirnerschütterungen?




Posttraumatische Amnesie (PTA): Schlag verursacht Koma → gelangt er das Bewusstsein
wieder, folgt Verwirrung
Retrograde Amnesie bzgl. der Ereignisse kurz vor dem Schlag
Anterograde Amnesie bzgl. Ereignisse während Verwirrung
Gedächtnisinseln: Erinnerungen an einzelne Ereignisse, die während diesen Perioden stattfanden
Nennen Sie drei Gedächtnisstrukturen (Schläfenlappen) und ihre mnemonischen Funktionen.








Hippocampus
Konsolidierung des LG für räumliche Beziehungen, nicht Speicherung selbst
Amygdala
Erinnern die emotionale Bedeutung von Erfahrungen
Riechhirn
Bildung des expliziten Langzeitgedächtnisses für Objekte
Inferotemporaler Cortex
Speicherung der Inhalte des LG
Cerebrellum
Implizite Erinnerungen des sensomotorischen Lernens
Präfrontaler Kortex
Gedächtnis der zeitlichen Abfolge von Ereignissen
Mediodorsaler Nucleus des Thalamus
Schädigung führt vermutlich zu Korsakow-Syndrom
Basales Vorderhirn
Beitrag zum Gedächtnis umstritten; bei Alzheimer immer geschädigt
Was ist eine „verzögerte Vergleichsaufgabe“? Welche Gehirnteile werden dabei beansprucht?




Testaufgabe, bei der einem Versuchstier ein bestimmtes Testobjekt vorgeführt wird, unter dem
sich eine Belohnung befindet. Nach einer Verzögerung sieht das Tier das ursprüngliche und ein
unbekanntes Objekt. Nun befindet sich die Belohnung unter dem neuen Objekt, Um sich in
diesem Test richtig zu verhalten, muss es das ursprüngliche Objekt erkennen und das unbekannte
wählen. → bei jedem Durchgang werden neue Objekte benutzt
→ist Verzögerungsintervall nur wenige Min. lang oder kürzer, verhalten sich nach Training,
gesunde Affen in 90% der trails richtig
Bei der verzögerten Vergleichsaufgabe werden Hippocampus, die Amygdala und das Riechhirn
(entorhinaler und perirhinaler Kortex) (alle im medialen Temporallappen) beansprucht
Läsionen des Riechhirns verursachen schwere Ausfälle in der Aufgabe; Läsionen des
Hippocampus und Amygdala nur geringe
Hirnschädigungen durch Ischämie: Rolle des Hippocampus bei Objekterkennung


Zusammenhang zw. Hippocampusschädigung bei Ischämie und Defiziten in der verz.
Vergleichsaufgabe → Hippocampus spielt doch wichtige Rolle bei Objekterkennung
Durch Ischämie bedinget Hyperaktivität der Pyramidenzellen der CA1-Region des Hippocampus
schädigt Neurone außerhalb des Hippocampus (durch hohe Aminosäureausschüttung) → diese
Schädigungen sind für Gedächtnisdefizite verantw.
Beschreiben Sie den Fall H.M.! Welche Gehirnverletzung liegt bei ihm vor? Welche
Erkenntnisse verdankt ihm die Gedächtnisforschung?







H.M. litt über zehn Jahre hinweg ab starken epileptischen Anfällen mit Zentrum im
mediotemporalen Cortex. Bilaterale Lobektomie des mediotemporalen Cortex inklusive des
Hippocampus und der Amygdala. →geringfügige retrograde Probleme, schwere anterograde
Probleme mit explizitem Gedächtnis (v.a. episodisches); Sein Kurzzeitgedächtnis war dagegen
intakt; es fehlte ihm aber jede Möglichkeit, Fakten ins Langzeitgedächtnis zu übertragen
(anterograde Amnesie)
Beitrag:
Widerlegte Prinzip der Äquipotenz → medialer Temporallappen spielt eindeutig wichtige Rolle
Einbeziehung des Hippocampus in mnestische Strukturen
Lobektomie löschte Fähigkeit zum Aufbau von Langzeiterinnerungen, ohne KG einzuschränken →
zwei untersch. Arten von Speicherung
Medialer Temporallappen spielt Rolle bei Konsolidierung, da KG intakt und keine retrograde
Amnesie
Bewies, dass es implizites Gedächtnis gibt (verbesserte Leistung, obwohl er sich nicht explizit an
Erfahrung erinnern kann)
Nenne zwei frühere Theorien zur Gedächtnisspeicherung!

Massenaktionsprinzip und Prinzip der Äquipotenz: Erinnerung werden im ganzen Neocortx
gespeichert und alle Teile des Neocortex haben die gleiche Bedeutung für Speicherung

Konsolidierungstheorien: Kurzzeitgedächtnis → Langzeitgedächtnis: Konsolidierung durch
Reverberationskreise (bewusstes Daran-denken) → Erfahrungen, die nicht lange genug im
Kurzzeitgedächntnis waren, können nicht ins LG gelangen
Nenne Tests, mit denen H.M. untersucht wurde!








Digit-span + 1-Test: 5 Ziffern wiederholen, beim nächsten Durchgang eine mehr → gesunde bis
zu 15 in 25 trails; H.M. nur 6
Corsi-Würfeltest: nonverbaler KG-Test: Vl berührt 9 Würfel in best. Reihenfolge → Patient soll
nachmachen → H.M. nur 5
Übereinstimmungstests: vorhergesehenes Objekt aus mehreren Objekten raussuchen →
Sequenz von Konsonanten möglich; Elipsenformen schlechter → Buchstaben besser im KG zu
wiederholen als Elipsen
Spiegelzeichnen: Hand verdeckt → über Spiegelbild Linie nachzeichnen → Leistungen werden
von mal zu mal besser, obwohl er sich nicht dran erinnert
Rotary-pursuit-Test: Zeiger auf rotierendes Objekt richten → Leistungen verbesserten sich ohne
Erinnerung → eventuell LG für sensomotorische Aufgaben
Gollins-Test: 5 Sets mit fragmentarischen Zeichnungen → wenn nicht erkannt: das nächst
vollständigere → Verbesserung ohne Erinnerung
Test zum Sprachverständnis → kaum Auffälligkeiten; Schwierigkeit bei mehrdeutigen Sätzen
Pawlowsche Konditionierung: nach Ton → Luftstoß ins Auge → H.M. blinzelt schon beim Ton
auch 2 Jahre später
Pawlowsche Konditionierung bei medialer Temporallappen-Amnesie


Verzögerte Konditionierung (UCS während CS noch vorhanden) klappt, da implizites Gedächtnis
gefordert
Spurenkonditionierung (UCS nach CS) nicht, da explizites Gedächtnis beteiligt
Fall R.B.

Ischämie nach Bypass-OP → Hirnschädigung v.a. im Hippocampus in Pyramidenzellenschicht →
Amnesie
Was versuchte man mit Elektrokonvulsionsschocks zu messen?



ECS: intensive, kurze Stromstöße, die dem Gehirn verabreicht werden, und Krampfanfall
bewirken → zur Behandlung schwerer Depressionen, Nebenwirkung: posttraumatische Amnesie
Theorie: ECS löscht durch Unterbrechung der neuronalen Aktivität alle Erinnerungen, die noch
nicht in strukturellen synaptischen Veränderungen manifestiert sind → Länge der
Konsolidierungsphase sollte bestimmt werden
Problem: es werden auch retrograde Amnesien von mehreren Wochen ausgelöst →
Unwahrscheinlich, dass Konsolidierung durch Aktivität der zirkulierenden Schaltkreise
(Reverberationskreise) gesteuert wird
Was ist Referenzgedächtnis und Arbeitsgedächtnis? Wie werden sie bei Ratten gemessen?






Referenzged.: für allg. Regeln und Fertigkeiten, die für eine Aufgabe benötigt werden
Arbeitsged.: temporäres Gedächtnis für Bewältigung einer gerade auszuführenden Aufgabe
Getestet mit radialem Labyrinth: mehrere Arme gehen von zentral gelegener Startkammer aus
→ täglich werden immer die gleichen Arme mit Futter belegt
→ Fähigkeit, nur diese aufzusuchen → Referenzged.
→Fähigkeit, jeden Arm nur einmal aufzusuchen → Arbeitsged.
Ratten mit Hippocampusläsionen haben Defizite in beiden
Nenne zwei Tests für das Raumgedächtnis bei Ratten!


Morrisches Wasserlabyrinth: Ratten werden an versch. Orten eines Beckens mit trübem
Wasser ausgesetzt und lernen auf eine Plattform kurz unter der Oberfläche zu schwimmen →
Hippocampusläsionen führen zu Defiziten
Radiales Labyrinth: s.o.
Wo liegt der Hippocampus? Wie heißen die Zellen, die aktiviert werden, wann man sich an
bestimmten Orten aufhält?


Der Hippocampus ist der medial gelegene Teil des Telencephalons und gehört zum limbischen
System.
Die Pyramidenzellen des Hippocampus sind sog. Ortszellen. (D. h., Neurone feuern nur dann,
wenn man sich an einem best., bereits bekannten Ort befindet, also in den „Ortsfeldern“ des
betreffenden Neurons.)
Was besagt die Theorie der kognitiven Landkarte?



Mehrere Systeme im Gehirn, die sich auf best. Gedächtnisinhalte spezialisieren
Hippocampus für Speicherung von Info über räumliche Beziehungen
Er erstellt allozentrische Landkarten der äußeren Welt und speichert diese (Präsentation des
Raums mit Beziehung zw. externen Objekten und Hinweisreize; nicht eigene Position als
Ausgangspunkt: egozentrisch)
Kapitel 15
Nennen Sie die Phasen der Neuralentwicklung!






Jede Zelle muss sich differenzieren, zum Bestimmungsort wandern, sich mit anderen Zellen
zusammenschließen und geeignete funktionale Verbindungen zu anderen Zellen aufnehmen
Bewältigung dieses Programms:
Induktion der Neuralplatte: 3 Wochen nach Empfängnis; Neuralplatte wird sichtbar → faltet sich
ein → bildet Neuralrinne → Ränder verschmelzen → Neuralrohr (Neurulation) → Hohlraum wird
zu Zentralkanal und Ventrikeln; nach 40 Tagen: Auftreibungen erkennbar aus denen sich Vorder-,
Mittel- und Rautenhirn entwickeln werden; Verlust der Totipotenz der Zellen (kann also nicht mehr
zu beliebigem Zelltyp werden)
Neurale Proliferation: Vermehrung der Zellen des Neuralrohrs; findet nicht gleichmäßig in allen
Bereichen statt (→artspez. Form des Gehirns); v.a. in der Ventrikularzone
Migration und Aggregation: Wanderung der neu entstandenen Nervenzellen zum
Bestimmungsort; Einpassen in Verband anderer Zellen im gleichen Areal, um Strukturen des NS
aufzubauen
Axonales Wachstum und Synapsenbildung: Zuwachsen der neuronalen Projektionen auf
geeignete Zielorte → Wachstumskegel an Spitze der wachsenden Fortsätze → Theorien: s. unten
Erläutern Sie den Einfluss der Erfahrung auf neuronale Entwicklung anhand eines Beispiels !


Erfahrung fördert die Entwicklung von aktiven neuronalen Verbindungen. Werden Ratten zum
Beispiel im dunkeln groß gezogen, sie also keinerlei Erfahrungen mit komplexen visuellen Stimuli
machen, bilden sich weniger Synapsen und weniger dendritische Dornen im primären visuellen
Kortex
Erfahrung kann aber auch in Konkurrenz zur Entwicklung treten: bleibt bspw. Nur ein Auge in
einer frühen Entwicklungsphase ohne Input, so wird die Sehfähigkeit dieses Auges blockiert; dies
ist jedoch nicht der Fall, wenn beide Augen abgedeckt sind
Wie sterben Nervenzellen?

Apoptose (prgrammierter Zelltod):
Zelltod durch genetisch determiniertes Selbstmordprogramm bei Fehlen von Neurotrophinen →
DANN und andere Strukturen werden in Membranen gepackt und dann von der äußeren
Zellmembran zersetzt (→ keine Entzündungen) → Synapsenneuanordnung; bei Hemmungen
dieses Programms → Krebs

Zellnekrose:
Zugrundegehen aufgrund von Verletzungen → Zellen platzen und verstreuen Inhalt in
extrazelluläre Flüssigkeit → Entzündungen. Immunzellen wandern ein und vertilgen die Zelle
Das zentrale Nervensystem entwickelt sich aus einem Neuralrohr, das wiederum im
Embryonalstadium aus der Neuralplatte hervorgeht. Derzeit werden drei Modelle diskutiert, die
auf unterschiedliche Art und Weise erklären, wie während des Neuronenwachstums Axone und
Dendriten den Weg zu ihren Bestimmungsorten finden. Beschreiben Sie diese Modelle und die
Befunde, die sie stützen. 3 Punkte

Chemoaffinitäts- Hypothese
Jede postsynaptische Oberfläche im NS setzt eine spezielle chemische Substanz frei (Marker),
von der best. wachsende Axone während der Entwicklung angezogen werden

Sperry durchtrennte die Sehnerven von Fröschen, drehte Augäpfel um 180° und
wartete, bis sich d. retinatalen Ganglienzellen, die d. Sehnerv bilden, regeneriert hatten
Zeigt danach Frosch Köder. War der Köder hinter dem Frosch schnappte er nach vorn →visuelle
Welt war ebenso wie Augen um 180° gedreht. Gleiches Ergebnis auch bei Fröschen, bei denen
Augen invertiert, aber d. Sehnerv nicht durchtrennt wurde.

Wegweiserneuronen- Hypothese
im unreifen Nervensystem gibt es bestimmte chemische o. mechanische Spuren, denen die
auswachsenden Axone zu ihren Bestimmungsorten folgen.
Die Pionier-Wachstumskegel finden die richtige Spur dadurch, dass sie mit neuronalen
Zelladhäsionsmolekülen) von Zellen (Wegweiserneuronen entlang des Weges Kontakt
aufnehmen.
Nachfolgenden Wachstumskegel orientieren sich dann an Spur, die von Pionierfasern
markiert ist (Fasciculation)

Werden bspw. Pionieraxone im Fischrückenmark zerstört, erreichen nachfolgende Axone ihren
normalen Zielort nicht.

Hypothese vom topographischen Gradienten
Axonales Wachstum ist von der relativen Position der Zellkörper zu sich überschneidenden
Gradienten bestimmt, nicht von einer Punkt-zu-Punkt- Zuordnung der Nervenverbindungen

Sehnerv erwachsener Frösche wurde durchtrennt und das Muster seiner Regeneration nach
Zerstörung v.Teilen der Retina oder d. optischen Tectums untersucht → Axone wuchsen nicht zu
ihren ursprünglichen Zielorten aus, sie wuchsen stattdessen so, dass um den verfügbaren Raum
in geordneter Weise auszufüllen → Axone, die vom restlichen Teil d. lädierten Retina
auswuchsen, breiteten sich geordnet aus, um den gesamten Raum des intakten Tectums
auszufüllen; Axone, die von einer intakten Netzhaut auswuchsen, quetschten sich zu einem
geordneten Muster zusammen, um den verbleibenden bereich des Tectums auszufüllen
Nennen Sie drei neuroplastische Reaktionen des NS auf Verletzungen!



Neuronale Degeneration: bei Durchtrennen von Axonen
Neuronale Regeneration: Wiederauswachsen geschädigter Neurone; im PNS fördern
Schwannzellen Regeneration von Axonen
Neuronale Reorganisation
Kapitel 16
Was versteht man unter Lateralisierung von Funktionen?



Linke und rechte Großhirnhemisphären bis auf cerebrale Kommissuren vollständig voneinander
getrennt
Lateralisierung: ungleiche Repräsentation psychologischer Funktionen in beiden
Großhirnhälften
Linke = dominante Hemisphäre, da dominierende Rolle bei komplexen Verhaltens- und
Kognitionsprozessen
Was wurde bei Split-Brain-Patienten durchgeführt?

Kommissurotomie: Durchtrennung der cerebralen Kommissuren
Raederscheidt (dt. Maler) litt am Neglect-Syndrom. Was versteht man darunter? Welche
Regionen des Gehirns sind bei Neglectpatienten geschädigt? (2 Pkt.)



Eine Verhaltensstörung, die sich als eine halbseitige Vernachlässigung bzw.
Aufmerksamkeits- und Wahrnehmungseinschränkung einer Raum- und Körperhälfte zeigt.
Personen mit dieser Störung reagieren nicht mehr auf visuelle, auditorische und
somatosensorische Reize an der zur Hirnschädigung kontralateralen Seite.
Ursache: Schlaganfälle,
Tritt kontralateral nach Schlaganfällen, Blutungen oder Tumoren auf, die (meist) den rechten
posterioren Parietallappen schädigen
Nennen Sie drei Tests zur Sprachlateralisierung!

Natriumamytaltest (Wada-Test)
Injektion einer kleinen Menge Natriumamytal in die linke oder rechte Halsschlagader →Betäubung
der ipsilateralen Hemisphäre → Test der Funktion der kontralateralen Hemisphäre (Aufsagen
bekannter Wortfolgen; gängige Objekte benennen); anschließend umgekehrt
→ in der Regel tritt Stummheit und Probleme bei Betäubung der linken Hälfte auf

Dichotischer Hörtest
Vpn hört über Kopfhörer 3 gesprochene Zahlenpaare, wobei jeweils eine Zahl auf das linke und
eine auf das rechte gegeben wird → die meisten erinnern sich besser an die Zahlen, die aufs
rechte Ohr gegeben wurden → Sprachdominanz der linken Hälfte, kontralaterale Verbindungen
setzen sich durch

Funktionelle Bildgebende Verfahren
Während Vpn liest, wird Hirnaktivität mittels PET oder fMRI aufgezeichnet →meist größere
Aktivität in der linken H.
In welchen Bereichen sind die beiden Hemisphären jeweils dominant?


Links: Sprache, Feinmotorik, Kontrolle ipsilateraler Bewegungen, verbales Gedächtnis, Bedeutung
von Erinnerungen
Rechts: räumliches Vorstellungsvermögen, Emotionalität, musikalische Fähigkeiten, nichtverbales Gedächtnis, Wahrnehmungsaspekte von Erinnerungen
Erläutern Sie zwei Theorien zur Asymmetrie der Gehirnhälften !

Analytisch-synthetische Theorie
es gibt zwei grundlegende Denkweisen: logisch-analytisch (links) vs. synthetisch (rechts)

Motorische Theorie
linke Hirnhälfte ist nicht auf Kontrolle der Sprache an sich spezialisiert, sondern für feinmotorische
Bewegungen im allg.. Gestützt wird die Theorie von der Tatsache, dass mit Läsionen die zu
Aphasie führen auch motorische Defizite einhergehen.

Linguistische Theorie: Der Patient W.L.
Die primäre Bedeutung der linken Hirnhälfte liegt in ihrer sprachlichen Fähigkeit
W.L. war taubstumm, konnte sich aber mittels Gebärdensprache verständigen. Nach einem
linkshemisphärischem Schlaganfall verlor er die Fähigkeit, die Zeichensprache zu produzieren
und zu verstehen → aber er war in der Lage komplexe Gesten zu produzieren und zu verstehen
→ Zeichensprache-Aphasie keine Folge motorischer oder sensorischer Defizite
Sprache wird durch ein Netzwerk neuronaler Strukturen in der Nähe der Fissura Lateralis
realisiert. Drei dieser Strukturen weisen Größenunterschiede zwischen linker und rechter
Hemisphäre auf. Nennen Sie diese drei Regionen und beschreiben Sie deren Lokalisation im
Gehirn. Welche Rolle spielen sie bei der Sprachverarbeitung? 3 Punkte

Frontales Operculum
Liegt im inferioren präfrontalen Kortex. In der linken Hemisphäre (Brodmann 44 and 45) liegt
hier das Broca-Areal
Ist für die Sprachproduktion und Artikulation essentiell

Planum temporale
Liegt im Temporallappen im hinteren Bereich der Fissura lateralis (Brodmann 22). Auf der linken
Hemisphäre liegt hier das Wernicke-Areal
Spielt eine Rolle beim Sprachverständnis

Heschl-Querwindung
auf Temporallappen in der Sylvischen Furche vor Planum temporale →Sitz des primären
auditorischen Kortex
Nimmt lautliche Stimuli auf (Lautstärke, Tonqualität)
Nennen Sie die typischen Symptome, die nach einer unilateralen Läsion im Bereich des
posterioren parietalen Kortex auftreten. Unter welchem Oberbegriff werden diese
Beeinträchtigungen subsumiert?

Ist die Läsion rechtshemisphärisch, kommt es meist zu kontralateralem Neglect. Dies
bezeichnet eine Störung, die dadurch charakterisiert ist, dass Reize aus der kontralateralen
Raum- und Körperhälfte nicht mehr beachtet werden. Außerdem kann es zu konstruktiver
Apraxie kommen, einer bilateralen Störung von Bewegungen, mit denen ein Objekt aus seinen
Bestandteilen zusammengesetzt werden muss.

Ist die Läsion linkshemisphärisch, kommt es zu einer Apraxie. Dabei handelt es sich um eine
Störung, bei der der Patient Schwierigkeiten hat, willkürliche Bewegungen auszuführen, obwohl
sie ihm leicht gelingen, solange er sie spontan in natürlichen Situationen ausführt.
Was passiert, wenn ein „Split-Brain-Patient“ aufgefordert wird, ein Objekt zu benennen, das in
seinem rechten bzw. linken Gesichtsfeld dargeboten wird? (Erkenntnisse von Sperry/
Gazzangia: Split-Brain-Patienten: rechte und linke Hemisphäre unabhängig voneinander.)



Wird das Objekt dem linken Gesichtsfeld (rechte H.) dargeboten, ist der Patient (linke
Hemisphäre) nicht in der Lage es zu benennen, kann jedoch mit der linken Hand (rechte
Hemisphäre) das richtige Objekt identifizieren.
Wird das Objekt rechts (linke H.) dargeboten, kann der Patient sagen, was es ist (Linke H.), es
mit der rechten Hand identifizieren, aber er kann es jedoch nicht mit der linken Hand wieder
erkennen (rechte H.).
Genauso, wenn Objekt in Hand gegeben wird
Was ist Cross-Cuing?



Nicht-neuronale Kommunikation der beiden Hemisphären bei Split-Brain-Patienten
Exp.: grüner oder roter Stimulus im linken Gesichtsfeld (rechte H.) → Patient soll Farbe nennen →
nach mehreren Versuchen verbesserte sich Leistung
Strategie: wenn rechte Hemisphäre das rote Licht sah und hörte, wie die linke Hemisphäre grün
sagte, reagierte sie, da sie wusste, dass Antwort falsch ist, mit Stirnrunzeln und Kopfschütteln →
dies gab linker Hemisphäre den Hinweis, dass die Antwort falsch war, und sie sich korrigieren
sollte
Was ist das helfende Hand Phänomen?
 Wenn eine Hand von Split-Brain-Patienten die andere umdirigiert
 Exp: Stift wird in linkem, Orange im rechten Gesichtsfeld präsentiert → Vpn soll Objekt
heraussuchen → rechte hand greift nach Orange (linke H.) → linke Hand (rechte H.) dirigiert
rechte Hand zum Stift
Was zeigt der Gesichtsschimären-Test?
 Zeigt die Fähigkeit beider Hemisphären eines Split-Brain-Patienten, gleichzeitig und unabh.
voneinander, Bilder zu vervollständigen
 Exp.: Fotos von Gesichtern, aus zwei Gesichthälften zusammengesetzt → sollte Patient
beschreiben, was er sah: vollständige Version der Gesichtshälfte im rechten Gesichtsfeld → sollte
er richtiges Gesicht unter mehreren Möglichkeiten wählen: Wahl des vollständigen Version der
Gesichtshälfte im linken Gesichtsfeld
Was ist die Z-Linse?
 Eine Kontaktlinse, deren eine Hälfte undurchsichtig ist
 Ermöglicht, dass nur eine Hemisphäre unabh. von der Augenbew. einen visuellen Input erhält (bei
Split-Brain-Patienten)
Erläutern Sie die Bestandteile des Wernicke-Geschwind-Modells und erklären Sie welche
Komponenten beim Unterhalten / bei lautem Lesen aktiviert werden und welche Funktionen Sie
haben !
 Serielles Modell, da jeder Prozess eine lineare Reaktionskette darstellt
 Alle in der linken Hemisphäre:
primärer visueller Cortex
primärer motorischer Cortex
primärer auditorischer Cortex
Broca-Areal (Sprachproduktion)
Wernicke-Areal (Verständnis)
Gyrus Angularis (Zentrum für Lesen)
Fasciculus arcuatus (Verarbeitung von Sprache, v.a. bei Wiederholungen gehörter Wörter)

Unterhaltung: Auditorische Signale werden vom primärer auditorischer Cortex empfangen →
weitergeleitet zum Wernicke-Areal, dort verstanden → Bei der Antwort generiert das WernickeAreal die neuronale Repräsentation des Gedankens → das Signal wird über den Fasciculus
arcuatus zum Broca-Areal übermittelt → dort wird das Artikulationsprogramm aktiviert → das die
Neuronen des primärer motorischer Cortex → und schließlich die Artikulationsmuskulatur
aktiviert

Laut Lesen: Gelesene Signale werden vom primären visuellen Kortex empfangen → zum
Gyrus Angularis geleitet → der die visuelle Form des Wortes in einen auditiven Code übersetzt
→ der zum Wernicke-Areal weitergeleitet und dort verstanden wird → Das Wernicke-Areal löst
die passende verbale Reaktion über den Fasciculus arcuatus im Broca-Areal aus, → das den
primären motorischer Cortex und schließlich die Artikulationsmuskulatur aktiviert

Moderne Untersuchungen mit bildgebenden Verfahren haben ergeben: Aussagen zum WernickeAreal scheinen zuzutreffen, wenn umgebende Gebiete einbezogen werden; Aussagen zum BrocaAreal und zum fasciculus arcuatus nicht haltbar; Gyrus angularis unklar
Erläutern Sie das Zweibahnen-Parallel-Modell!






Beruht aus Untersuchungen zur Dyslexie
Lautes Lesen wird simultan von über einen lexikalischen und einen nicht-lexikalischen
Kanal vermittelt
Lexikalische Sprachverarbeitung basiert auf gespeicherter Info über geschriebene Worte
Nicht-lexikalische Sprachverarb. basiert auf allg. Regeln der Aussprache und erlaubt auch
unbekannte oder Nicht-Wörter auszusprechen
Oberflächendyslexie: Störung des lexikalischen Wegs; nicht-lexikalischer intakt → Wörter
können nur noch dann ausgesprochen werden, wenn sie den normalen Ausspracheregeln
entsprechen
Tiefendyslexie: lexikalischer Weg intakt; Störung des nicht-lexikalischen Wegs → können nur
noch bekannte Wörter aussprechen
Aphasien zählen zu den häufigsten Folgen von Schlaganfällen. Welche Unterformen der
Aphasie kennen Sie? Wodurch sind sie gekennzeichnet?

Aphasie: durch Hirnschädigung verursachte Störung der Fähigkeit Sprache zu verstehen oder zu
produzieren

Globale Aphasie
In diesem Fall sind alle sprachlichen Bereiche stark beeinträchtigt. Dazu gehören spontanes
Sprechen, Nachsprechen, Verstehen, Lesen und Schreiben.

Broca- Aphasie (hypothetisch)
Defizite primär expressiver Natur → Die Sprachproduktion bei Menschen mit Broca-Aphasie ist
erheblich verlangsamt und mit erheblicher Sprechanstrengung und undeutlicher Artikulation
verbunden. Die grammatische Struktur der Sätze ist auf einzelne kommunikativ wichtige
Substantive, Verben und Adjektive reduziert. Das Sprachverständnis ist nur relativ gering gestört.
→ selektive Läsionen des Broca-Areals

Wernicke- Aphasie (hypothetisch)
Defizite primär rezeptiver Natur → Verständnis für geschriebene und gesprochene Sprache ist
beeinträchtigt; Sprache verliert Bedeutung, weist aber oberflächlich betrachtet Struktur, Rhythmus
und Intonation normaler Sprache auf (Wortsalat) → selektive Läsion des Wernicke-Areals

Leitungsaphasie (hypothetisch)
Sprchverständnis und Spontansprache intakt; Schwierigkeiten beim Wiederholen gehörter Wörter
→ Schädigung des Fasciculus arcuatus (Nervenbahn, die Broca- und Wernicke-Areal verbindet)

Amnestische Aphasie
Die Amnestische Aphasie ist eher eine Wortfindungsstörung. Das Sprachverständnis ist meisten
kaum gestört. Allerdings ist die Rede häufig geprägt durch geringe Vermittlung von Inhalten.
Häufig werden gesuchte Wörter durch Füllwörter ersetzt.
Stress
Wie wirken Glucocorticoide im ZNS und PNS?
Glucocorticoide sind Steroidhormone, die als Stressreaktion aus Nebennierenrinde (adrenalen Cortex)
ausgeschüttet werden. Bei Kurzzeitstress verhindern sie Übermaß an peripheren und zentralnervösen
Reaktionen
Im ZNS beeinflussen sie die sensorische Infoverarb. → Wahrnehmungsschwelle nimmt zu
(Reize müssen intensiver sein) → gesenkte Erregbarkeit; Schlaflosigkeit, Euphorie
Im PNS wirken sie: akustisches System: Stapediusreflex (Dämpfung der Druckübertragung bei
lauten Tönen durch Versteifung der Steigbügel) wird durch Kortisol gehemmt → hebt
Wahrnehmungsschwelle für intensive Töne an → schützt vor weiterer Erregung
Schildern Sie den “circulus vitiosus“ zwischen Altern, Gedächtnis und Stress


Dichte der Rezeptoren für Glukokortikoide im ZNS besonders hoch in Bereichen des Limbischen
Systems, v.a. im Hippocampus. → Vermehrte Ausschüttung von Glukokortikoiden im Alter oder
nach chronischem Stress führt zu → Vernichtung von hippocampalen Neuronen und von deren
spezifischen Rezeptoren. → Dadurch 1. erhebliche Funktionseinschränkungen, z.B.
Verschlechterung der Gedächtnisses und 2. Verminderung der über die Rezeptoren vermittelten
Feedbackhemmung der Kortisolausschüttung → führt zu weiterer Vernichtung von
Hippocampusneuronen
bei sehr traumatischen Ereignissen → kortisolinduzierte Amnesie, um ständige Stressreizung zu
verhindern
Cortisol spielt eine wesentliche Rolle bei Stressreaktionen. Welche Wirkung hat es auf den
Organismus? Welche Funktion erfüllt es in Stressreaktionen? (2 Pkt.)

Cortisol hat im Stoffwechsel vor allem Effekte auf den Kohlenhydrathaushalt, den Fettstoffwechsel
und den Proteinumsatz. Cortisol wirkt immunologisch (immunsupressiv, enzündungshemmend,
antiallergisch). Es bewirkt gesenkte Erregbarkeit, Schlaflosigkeit und Euphorie.

Als Folge der Kortisolausschüttung in Stresssituationen steigt die Gerinnungsfähigkeit des Blutes
an und Entzündungs- und Immunreaktionen werden vermindert; die Erregbarkeit wird gesenkt.
Nennen Sie die 3 Reaktionsphasen des „Allgemeinen Anpassungssyndroms“ (Stressreaktion
nach Seyle) ! Wodurch sind diese charakterisiert?

Alarmreaktion
Der Organismus reagiert auf die akute Einwirkung des Stressors → kurzfristige Aktivierung aller
Alarmsysteme: Aktivierung des Sympathiko-NNM-Systems, Ausschüttung von Adrenalin und
Kortisol (entzündungshemmend), Steigerung des Energieumsatzes (bessere Leistungsfähigkeit),
Senkung des Testosteronspiegels, gesenktes Schmerzempfinden, Körpertemperaturabsenkung.
Symptome: Kopfschmerz, Müdigkeit, Appetitverlust

Widerstandsphase (bei konstantem Stressor)
Körperliche Anpassung: Die Skelettmuskeln werden verstärkt mit Sauerstoff und Nährstoffen
versorgt, Steigerung des Blutdruckes, Verstärkung der Atmung, Freisetzung von Fettsäuren aus
dem Fettgewebe, die Darmmuskulatur wird entspannt, die Verdauung gehemmt. Erhöhte
Hormonausschüttung, insbes. ACTH. Hält die Stresssituation länger an, kommt es zu
gegenregulatorischen Wirkung des Parasympathikus

Erschöpfungsphase
die adaptive Kapazität geht verloren. Es kommt zu Energiebereitstellungsproblemen, die
Wachstums-, Fortpflanzungsprozesse und die Immunabwehr funktionieren nicht mehr. Kann zum
Zusammenbrechen des Organismus oder zum Tod führen
Stelle die Unterschiede zwischen subdominanten und submissiven Verlierern dar
(verhaltensbezogen und physiologisch)


subdominante Verlierer: sind überaktiv, vermeiden die Konfrontation, beobachten ständig,
verteidigen sich aber bei einer Konfrontation
Steigerung der Herzschlagrate; Tag-und-Nacht-Rhythmus fällt weg; Aktivierung des SympathikoNNM-Systems (→ führt nicht zum Tod)
submissiven Verlierer: verkriechen sich, Attacken werden meist ohne Gegenwehr ertragen, das
Putz- und Komfortverhalten wird eingestellt, sie vermitteln dem Menschen einen apathischen,
depressiven Eindruck. → bei Trennung von siegreichem Männchen: keine Symptome; beim
bloßen Anblick: Stress
Schilddrüsenhormone nehmen ab → keine Energiebereitstellung mehr, Gesamteiweiß sinkt, mehr
Harnstoff, rote Blutkörperchen zerfallen.
Zunahme von Glukokortikoiden und Cholesterol
Nenne die zwei Systeme der Stressreaktion!


Sympathiko-Nebennierenmark-System: Stressor aktiviert über sympathisches NS die
Ausschüttung von Adrenalin und Noradrenalin aus NNM → bei kurzfristigen Stressreaktionen
Hypothalamus-Hypophysenvorderlappen-Nebennierenrinde: bei längerer Aktivierung des
Sympathiko-NNM-Systems → Adrenalin bewirkt, dass Hypothalamus Kortikotropin-ReleasingHormon (CRH) ausschüttet → HVL schüttet daraufhin Adrenokortikotropes Hormon (ACTH) aus
→ dies wiederum wirkt auf NNR → NNR schüttet Glukokortikoide (Kortisol) aus
→ bei langfristigen Stressreaktionen (dient auch Stressbewältigung, da bei Erwartung schon
vorher aktiviert werden kann → verringert Stressreaktion)
Was versteht man unter gelernter Hilflosigkeit? Experimentelle Anordnung?





Entkommt man einem Stressor lange nicht, ergibt man sich, selbst wenn dann die Möglichkeit zur
Flucht besteht
Triadischer Versuchsplan:
Phase I: Gr.1: Käfig mit zwei Hälften → eine davon unter Strom → Hund kann über Barriere
springen; Gr.2: keine Möglichkeit, dem Stressor zu entkommen
Phase II: Konditionierung: wenn Lampe leuchtet, dann keine Stromschläge in diesem Bereich
Erg.: nur Gruppe 1 ließ sich konditionieren; Gr.2: motor. und assoziative Defizite; Leerung der
Noradrenalinspeicher; Disregulation des Dopaminsystems
Nenne Faktoren, von der die Wirkung von Stressoren abhängt!







Stress: die physiologische Veränderungen aufgrund der Wirkung eines Stressors
Objektive Intensität des Stressors
Subjektive-psychologische Intensität
Coping (Bewältigungsstrategien)
Vorerfahrung
Persönlichkeit
Ausgangszustand (z.B. Hormonstatus (je mehr Kortisol im Blut, desto geringer die Wirkung))
Herz-Kreislauf-System
Was versteht man unter „respiratorischer Sinusarrhythmie“?
 Unter respiratorischer Sinusarrhythmie versteht man die Erhöhung der Herzfrequenz beim
Einatmen und die Verringerung der Herzfrequenz beim Ausatmen
Was ist der PQRST-Komplex?
 Normales EKG-Muster
 Entstehung:
 P-Welle: Erregung der Vorhöfe
 PQ-Strecke: komplette Erregung
 Q-Zacke: Erregung der Kammerscheidewand
 R: fortlaufende Erregung der Kammerwand
 S: Erregung der Kammerseitenwände
 ST-Strecke: komplette Erregung der Kammerwände
 T-Welle: Abnahme der Erregung
Nenne die vier Phasen des Herzschlags!
 Kammerfüllung (passiver Vorgang)
 Anspannung
 Austreibung (→erste Vibration im Herzschlag)
 Erschlaffung (→zweite Vibration)
 Systole: Anspannung und Austreibung
 Diastole: Kammerfüllung und Erschlaffung
Psychophysiologie
Was ist das ARAS? Wo befindet es sich?

Das aufsteigende reticuläre Aktivierungssystem auch Formatio reticularis genannt. Es besteht aus
etwa 100 Kernen ist und ist an verschiedenen Funktionen wie Schlaf, Aufmerksamkeit,
Bewegung, Erhalt des Muskeltonus und verschiedene Herz-, Kreislauf- und Atemreflexe beteiligt.
Es zieht sich durch das Myelencephalon.
Unspezifische Aktivierungstheorien gehen von einer einheitlichen Aktivierungsrichtung aller
involvierten Teilsysteme aus. Die Theorien wurden allerdings bereits in den 60ern von Lacey
und Mitarbeitern kritisiert. Welches Alternativmodell hat Lacey formuliert? Durch welche
empirischen Befunde wird es gestützt?

Lacey entwickelte die Theorie der Fraktionierung der Aktivierungsrichtung. Sie besagt, dass
verschiedene Komponenten des autonomen Nervensystems je nach Situation, unter Aktivierung
gegenläufige Funktionsverschiebungen zeigen können

Reizaufnahme (environmental intake) bewirkt Herzfrequenzabfall und Herabsetzung des
Blutdrucks. Das Abblocken von Außenreizen (environmental rejection) hatte dagegen einen
Anstieg dieser beiden Parameter zur Folge.
Was besagt die Yerkes-Dodson-Regel?

Bei Unterforderung bleibt der Mensch hinter seinen Möglichkeiten zurück - es entsteht ein
Leistungsleck. Durch ein gesundes Maß an emotionaler Aktiviertheit kann die Leistung bis zu
einem Spitzenwert gesteigert werden. Erhöht sich das Erregungsniveau über das erforderliche
Maß, sinkt die Leistung wieder ab. Zwischen der physiologischen Aktivierung und der
Leistungsfähigkeit besteht ein umgekehrt u-förmiger Zusammenhang.
Was versteht man unter autonomer Balance?

autonome Balance ist das Gleichgewicht zwischen zwei antagonistischen Systemen
(Sympathikus; Parasympathikus)
Was ist Aktivierung?

Veränderung des Leistungsniveaus → Grundprozess des Organismus, der einer Optimierung der
psychophysischen Basis für adäquates Reagieren auf externe oder interne Anforderungen dient
Was versteht man im Rahmen der kassischen Konditionierung unter aktivierungsabhängiger
Verstärkung ?

Verstärkung der synaptischen Übertragung, die nur dann induziert wird, wenn das Neuron
während des verstärkungsinduzierten präsynaptischen Inputs aktiv ist. Man spricht auch
von einer synaptischen Bahnung, wichtig ist hierbei die zeitliche Aktivierung zwischen
US und CS: optimale Konditionierung nur wenn CS+ dem US 0,5 s vorausgeht
Aufmerksamkeit
Welche Defizite zeigen Neglectpatienten in der Posner cuing Aufgabe?



Bei Neglect Patienten findet man einen sign. Unterschied zwischen den RT auf kontraläsionalen
und ipsiläsionalen Stimulus in der „invalides Cuing“-Bedingung, den man bei gesunden Menschen
nicht findet. Bei gesunden Menschen nimmt RT bei invalidem Cuing gegenüber validem Cuing
immer zu, egal auf welcher Seite Stimulus gezeigt wird.
Neglectpatienten zeigen dagegen besonders verzögerte Reaktionen, wenn ipsiläsionales (das
beachtete) Halbfeld gecuet wurde, aber der Reiz im contraläsionalen (vernachlässigten) Halbfeld
erscheint (invalides Cuing)(er dachte gleich kommt der Reiz, aber sieht nichts). Wenn
kontraläsionales Halbfeld invalide gecued wurde, also der Reiz dann im ipsiläsionalen Feld
auftaucht, dann kein sign. Unterschied zur RT bei validem Cuing.
Grundsätzlich sind die RT verzögerter als bei gesunden.
Ordnen Sie die entsprechenden Begriffe einander zu:
Hypophyse
Gyri
limbisches System
Hypoxie
Thrombose/Thrombus
Diencephalon
Mesencephalon
Anterior
dorsal
superior
posterior
inferior
Meningion
Alzheimer-Krankheit
Kindling-Phänomen
Huntington-Krankheit
Hämatom
Multiple Sklerose
Infarkt
Encephalitis
Nucleus caudatus
Aquaeductus cerebri
Thalamus
Hirnanhangdrüse
Gehirnwindungen
Sauerstoffmangel im Blut
Pfropfen im Gefäß
Zwischenhirn umfasst Thalamus, Hypothalamus, Hypophyse etc.
Mittelhirn,
in Richtung Nasenspitze gelegen
rückenwärts
oben liegend
hinten liegend = dorsal
unten liegend = kaudal
Ein Tumor, der zwischen den Hirnhäuten wächst
Altersdemenz
Krämpfe nach wiederholter elektrischr Stimulation der Amygdala
Veitstanz, Störung motorischer und intellektueller Fähigkeiten
Bluterguss
Erkrankung durch Abbau des ZNS-Myelins
Eine Region mit ischämischer oder hämorrhagischer Schädigung
Gehirninfektionen
wichtige Kernstruktur in den Basalganglien
Verbindungskanal zwischen III. und IV Ventrikel
Kernpaare die in den Cortex projizieren...
Schreiben Sie die Abkürzungen aus:































EKP: Ereignis-korrelierte Potentiale,
EDA: Elektrodermale Aktivität
EEG: Elektroenzephalogramm
EOG: Elektrookulogramm
EKG: Elektro-Kardiogramm
fMRI/fMRT: Funktionelle Kernspintomographie (Functional
tomographie)
EMG: Elektromyogramm
BOLD: Blood Oxygenation Level Dependent
EPSP: exzitatorisches postsynaptisches Potential
IPSP: Inhibitorisches postsynaptisches Potential
2-DG: 2-Desoxyglucose
GABA: Gamma-Aminobuttersäure
ACh: Acetylcholin
PNS: peripheres Nervensystem
MEG: Magnetoenzephalographie
PET: Positronen-Emissions-Tomographie
ACTH: Adrenocorticotropes Hormon
AEP: Akustisch-evoziertes-Potential
SEP: somato-sensibel evozierten Potentiale
VEP: visuell evozierten Potentiale
OEP: olfaktorisch evozierten Potentiale
GEP: gustatorisch evozierten Potentiale
CRH: Corticotropes Releasinghormon
magnetic resonance imaging /
CT: Computertomographie
SCL: Hautleitwertniveau (skin conductance level)
SCR: Hautleitfähigkeitsreaktion (skin conductance reaction)
IP: intraperitoneal
IM: intramuscular
IV: intravenous
SC: subcutan
6-OHDA: 6-Hydroxydopamin
????
Räumliche Aktivitäten bei Mensch und Tier lassen sich in drei verschiedene Typen unterteilen.
Wie heißen diese räumlichen Reaktionen? Nennen Sie Anordnungen mit denen die
Reaktionsformen untersucht und voneinander abgegrenzt werden können. 3 Punkte fffffffffffff
Herunterladen